Nure 132 Endocrine Exam

अब Quizwiz के साथ अपने होमवर्क और परीक्षाओं को एस करें!

Angioedema

Localized areas of swelling beneath the skin, often around the eyes and lips, but it can also involve other body areas as well. (medical emergency)

Calcitonin

Lowers blood calcium levels

mannitol (Osmitrol)

Mannitol, a diuretic, can be used for the reduction of intracranial pressure and brain mass, to reduce intraocular pressure if this is not achievable by other means, to promote diuresis for acute renal failure to prevent or treat the oliguric phase before irreversible damage, and to promote diuresis to promote excretion of toxic substances, materials, and metabolites.

the main drug used to treat Acromegaly

Octreotide (Sandostatin) (it reduces GH levels to normal in many patients. It is given by subcutaneous injection three times a week.)

Drugs for Acromegaly

Octreotide (Sandostatin) dopamine agonists (when remission has not been achieved after surgery) GH antagonist (Reduce effective growth hormone in the body by blocking liver production of IGF-1. Most patients taking this drug achieve normal IGF-1 levels with symptom improvement

combination insulin therapy

-Can mix short- or rapid-acting insulin with intermediate-acting insulin in same syringe -Provides mealtime and basal coverage in one injection -Commercially premixed or self-mix -mealtime and basal coverage Disadvantage -most people achieve optimal blood glucose with basal-bolus therapy

Endocrine hormones; common characteristics

-Secreted in small amounts -Regulated by feedback systems -bind to specific receptors

Hypothalamus location and function

-a neural structure that directs several maintenance activities (eating, drinking, body temp), helps govern the endocrine system via the pituitary gland, and is linked to emotion and reward -located in the limbic system below (hypo) the thalamus

Endocrine Hormones

-control and regulate target cells or organs -released directly into circulation or have local effect

normal specific gravity range (Urine)

1.005 to 1.030

HA1C

A hemoglobin A1C (HbA1C) test is a blood test that shows what your average blood sugar (glucose) level was over the past two to three months. (has DM <7 screening for DM <6.5)

tropins

A hormone that induces the release of other hormones. (The pituitary gland releases tropins which cause other glands of the endocrine system to release hormones unique to those glands, often in a negative feedback relationship to the pituitary secretions.)

Describe the negative feedback in regulating hormone secretion

A negative feedback loop is one way that the endocrine system tries to keep homeostasis (stability) in the body. If an endocrine gland senses that there is too much of one hormone in the body, it will initiate changes to decrease production of that hormone

metabolic syndrome

A syndrome marked by the presence of usually three or more of a group of factors (as high blood pressure, abdominal obesity, high triglyceride levels, low HDL levels, and high fasting levels of blood sugar) that are linked to increased risk of cardiovascular disease and Type 2 diabetes.

During a visit to a community, the nurse will recommend routine screening for diabetes when the person has one or more of seven risk criteria. Which of the following persons that the nurse comes in contact with most needs to be screened for diabetes based on the seven risk criteria? A. A client with an HDL cholesterol level of 40 mg/dl and a triglyceride level of 300 mg/dl B. A woman who is at 90% of standard body weight after delivering an eight-pound baby C. A middle-aged Caucasian male D. An

A. A client with an HDL cholesterol level of 40 mg/dl and a triglyceride level of 300 mg/dl (The seven risk criteria include: greater than 120% of standard bodyweight, certain races but not including Caucasian, delivery of a baby weighing more than 9 pounds or a diagnosis of gestational diabetes, hypertensive, HDL greater than 35 mg/dl or triglyceride level greater than 250 or a triglyceride level of greater than 250 mg/dl, and, lastly, impaired glucose tolerance or impaired fasting glucose on prior testing.)

According to the National Diabetes Statistics Report, diabetes remains one of the leading causes of death in the United States since 2010. Which of the following factors are risks for the development of diabetes mellitus? Select all that apply. A. Age over 45 years B. Overweight with a waist/hip ratio >1 C. Having a consistent HDL level above 40 mg/dl D. Maintaining a sedentary lifestyle E. Polycystic ovary syndrome

A. Age over 45 years B. Overweight with a waist/hip ratio >1 D. Maintaining a sedentary lifestyle E. Polycystic ovary syndrome (Diabetes mellitus (DM) is a metabolic disease, involving inappropriately elevated blood glucose levels. The main subtypes of DM are Type 1 diabetes mellitus (T1DM) and Type 2 diabetes mellitus (T2DM), which classically result from defective insulin secretion (T1DM) and/or action (T2DM). Option A: Aging results in a reduced ability of beta cells to respond with insulin effectively. More than 25% of the U.S. population aged ?65 years has diabetes, and the aging of the overall population is a significant driver of the diabetes epidemic. Option B: Overweight with waist/hip ratio increase is part of the metabolic syndrome of DM II. T2DM involves a more insidious onset where an imbalance between insulin levels and insulin sensitivity causes a functional deficit of insulin. Insulin resistance is multifactorial but commonly develops from obesity and aging. Option C: The NCEP ATP III guidelines indicate that the ideal HDL levels should be between 40 and 60 mg/dL. The primary function of HDL is the transport of cholesterol from the peripheral tissues to the liver, playing a role in the biodistribution of lipids. HDL is known for its anti-atherogenic and anti-inflammatory properties, thanks to its uptake and return of the cholesterol stored in the foam cells of atherosclerotic plaques to the liver. Option D: There is an increase in atherosclerosis with DM due to metabolic syndrome and a sedentary lifestyle. Relatively high amounts of sedentary time (daily/weekly sitting time) have been associated with significantly greater risk for type 2 diabetes and metabolic syndrome in two meta-analyses. Option E: For women, having polycystic ovary syndrome — a common condition characterized by irregular menstrual periods, excess hair growth, and obesity — increases the risk of diabetes.

Billy is being asked concerning his health in the emergency department. When obtaining a health history from a patient with acute pancreatitis, the nurse asks the patient specifically about the history of: A. Alcohol use B. Cigarette smoking C. Diabetes mellitus D. High-protein diet

A. Alcohol use (Alcohol use is one of the most common risk factors for pancreatitis in the United States. In the majority of cases, alcohol use, gallstones, and hypertriglyceridemia cause acute pancreatitis. A thorough history regarding alcohol use and medications should be gathered, keeping in mind that over five years of heavy alcohol use is often needed to induce alcohol-related pancreatitis.)

When reviewing the urinalysis report of a client with newly diagnosed diabetes mellitus, the nurse would expect which urine characteristics to be abnormal? Select all that apply. A. Amount. B. Odor. C. pH. D. Specific gravity. E. Glucose level. F. Ketone bodies.

A. Amount. B. Odor. E. Glucose level. F. Ketone bodies. (Diabetes mellitus is associated with increased amounts of urine, a sweet or fruity odor, and glucose and ketone bodies in the urine. It does not affect the urine's pH or specific gravity. Option A: Excessive thirst and increased urination are common diabetes signs and symptoms. When the client has diabetes, excess glucose — a type of sugar — builds up in the blood. The kidneys are forced to work overtime to filter and absorb the excess glucose. Option B: Strong sweet-smelling urine is a sign of advanced diabetes, which can be diagnosed with urinalysis. With advanced diabetes, sugar and ketones, which are normally absent, can accumulate in the urine and create a strong odor. Option C: A diet that includes too many acid-producing foods, such as protein or sugar, can cause acidity in the urine as well as other negative health effects. This may cause a type of kidney stone called uric acid stones to form. Option D: In uncontrolled diabetic patients, the urine specific gravity might reach 1.045 to 1.050 as a result of the loss of glucose in the urine. Specifically, in a patient with poor glycemic control elevated blood glucose levels should result in an increased urine specific gravity. Option E: In diabetic patients, the kidneys are more susceptible to the effects of hyperglycemia; many of the kidney cells are unable to decrease glucose transport rates and unable to prevent intracellular hyperglycemia in an increased glucose concentration state. Option F: If cells don't get enough glucose, the body burns fat for energy instead. This produces a substance called ketones, which can show up in the blood and urine. High ketone levels in urine may indicate diabetic ketoacidosis (DKA), a complication of diabetes that can lead to a coma or even death.)

Nurse Shey is educating a pregnant client who has gestational diabetes. Which of the following statements should the nurse make to the client? Select all that apply. A. Cakes, candies, cookies, and regular soft drinks should be avoided. B. Gestational diabetes increases the risk that the mother will develop diabetes later in life. C. Gestational diabetes usually resolves after the baby is born. D. Insulin injections may be necessary. E. The mother should strive to gain no more weight during pregnancy. F. The baby will likely be born with diabetes

A. Cakes, candies, cookies, and regular soft drinks should be avoided. B. Gestational diabetes increases the risk that the mother will develop diabetes later in life. C. Gestational diabetes usually resolves after the baby is born. D. Insulin injections may be necessary. (Gestational diabetes can occur between the 16th and 28th week of pregnancy. If not responsive to diet and exercise, insulin injections may be necessary. Concentrated sugars should be avoided. Option A: Most carbohydrates are found in starchy or sugary foods. They include bread, rice, pasta, cereal, potatoes, peas, corn, fruit, fruit juice, milk, yogurt, cookies, candy, soda, and other sweets. Try to avoid eating simple carbohydrates, such as potatoes, french-fries, white rice, candy, soda, and other sweets. This is because they cause the blood sugar to rise quickly after eating such foods. Option B: Women with gestational diabetes (GDM) have an increased 35 to 60% risk of developing diabetes mellitus over 10 to 20 years after pregnancy. Gestational diabetes etiology is apparently related to 1) the pancreatic beta-cell dysfunction or the delayed response of the beta cells to the glycemic levels, and 2) the marked insulin resistance secondary to placental hormonal release. Option C: ADA and ACOG recommend repeating testing every 1 to 3 years for women who developed GDM and had normal postpartum screening results. At 4 to 12 weeks postpartum, the recommendation is to perform a 75g oral glucose tolerance test to rule out the possibility of the development of type 2 diabetes, impaired fasting glucose, or impaired glucose tolerance test. Option D: Insulin can help achieve an appropriate metabolic control, and it is added to the management if fasting blood glucose is greater or equal to 95 mg/dL, if 1-hour glucose level is greater or equal to 140 mg/dL, or if 2-hour glucose level is greater than 120 mg/dL. Option E: Weight gain should continue, but not in excessive amounts. The clinical features of gestational diabetes mellitus can be varied. The disproportionate weight gain, obesity, and elevated BMI can be suggestive features. The diagnosis is established by the laboratory screening method at the Option F: Usually, gestational diabetes disappear

Gregory is a 52-year-old man identified as high-risk for diabetes mellitus. Which laboratory test should a nurse anticipate a physician would order for him? Select all that apply. A. Fasting Plasma Glucose (FPG) B. Two-hour Oral Glucose Tolerance Test (OGTT) C. Glycosylated hemoglobin (HbA1C) D. Fingerstick glucose three times daily E. Urinalysis and urine culture

A. Fasting Plasma Glucose (FPG) B. Two-hour Oral Glucose Tolerance Test (OGTT) (When an older person is identified as high-risk for diabetes, appropriate testing would include FPG and OGTT. An FPG greater than 140 mg/dL usually indicates diabetes. The OGTT is to determine how the body responds to the ingestion of carbohydrates in a meal. Option A: Current laboratory recommendations for plasma glucose measurement are to draw fasting blood samples in the morning rather than later in the day, as glucose levels tend to be higher in the morning than in the afternoon. Option B: The OGTT requires a fasting blood glucose measurement in the morning. After the measurement, the patient receives oral glucose (usually a glucose load of 75g anhydrous glucose dissolved in water) that the patient consumes. The plasma glucose levels are measured again at 1-hour and 2-hours to analyze the glucose level changes. Option C: HbA1C evaluates long-term glucose control. The hemoglobin A1c (glycated hemoglobin, glycosylated hemoglobin, HbA1c, or A1c) test is used to evaluate a person's level of glucose control. The test shows an average of the blood sugar level over the past 90 days and represents a percentage. The test can also be used to diagnose diabetes. Option D: A fingerstick glucose three times daily spot-checks blood glucose levels. The use of glucose meters is common in physician offices or by patients to monitor blood glucose levels and establish patterns of glucose fluctuations over time with regular use and recording. Option E: Urinalysis and urine cultures, in which bacteria from a urine sample are grown in a laboratory, are done to diagnose a urinary tract infection. Urine should be cultured in all men and all patients with diabetes mellitus, who are immunosuppressed, and women who are pregnant. Classic teaching on urine culture sets the gold standard for infected urine at greater than 10 colony forming units (CFU).)

37. A female patient is scheduled for an oral glucose tolerance test. Which information from the patient's health history is important for the nurse to communicate to the health care provider regarding this test? a. The patient uses oral contraceptives. b. The patient runs several days a week. c. The patient has been pregnant three times. d. The patient has a family history of diabetes.

ANS: A (Oral contraceptive use may falsely elevate oral glucose tolerance test (OGTT) values. Exercise and a family history of diabetes both can affect blood glucose but will not lead to misleading information from the OGTT. History of previous pregnancies may provide informational about gestational glucose tolerance but will not lead to misleading information from the OGTT.)

After suffering an acute MI, a client with a history of type 1 diabetes is prescribed metoprolol (Lopressor) I.V. Which nursing interventions are associated with I.V. administration of metoprolol? Select all that apply. A. Monitor glucose levels closely. B. Monitor for heart block and bradycardia. C. Monitor blood pressure closely. D. Mix the drug in 50 ml of dextrose 5% in water and infuse over 30 minutes. E. Be aware that the drug is not compatible with morphine.

A. Monitor glucose levels closely. B. Monitor for heart block and bradycardia. C. Monitor blood pressure closely. (Metoprolol is a cardioselective beta-1-adrenergic receptor inhibitor that competitively blocks beta1-receptors with minimal or no effects on beta-2 receptors at oral doses of less than 100 mg in adults. It decreases cardiac output by negative inotropic and chronotropic effects. Option A: Metoprolol masks the common signs of hypoglycemia; therefore, glucose levels should be monitored closely in diabetic clients. The mechanism responsible for ?-blocker-induced hypoglycemia involves inhibition of hepatic glucose production, which is promoted by sympathetic nervous stimulation. In addition, adrenergic counter-regulation is diminished, resulting in a reduction in glycogenolysis. Option B: When used to treat an MI, metoprolol is contraindicated in clients with heart rates less than 45 beats/minute and any degree of heart block, so the nurse should monitor the client for bradycardia and heart block. Option C: Metoprolol masks common signs and symptoms of shock, such as decreased blood pressure, so blood pressure should also be monitored closely. Beta-blockers, including atenolol and metoprolol, may mask the signs of tachycardia and diaphoretic skin seen in patients in shock. Option D: The nurse should give the drug undiluted by direct injection. Lopressor, metoprolol tartrate USP, is a selective betai-adrenoreceptor blocking agent, available in 5-mL ampoule for intravenous administration. Each ampul contains a sterile solution of metoprolol tartrate USP, 5 mg, and sodium chloride USP, 45 mg, and water for injection USP. Option E: Although metoprolol should not be mixed with other drugs, studies have shown that it is compatible when mixed with morphine sulfate or when administered with alteplase infusion at a Y-site connection.)

Which of the following is accurate pertaining to physical exercise and type 2 diabetes mellitus? A. Physical exercise can slow the progression of type 2 diabetes mellitus. B. Strenuous exercise is beneficial when blood glucose is high. C. Patients who take insulin and engage in strenuous physical exercise might experience hyperglycemia. D. Adjusting insulin regimen allows for safe participation in all forms of exercise.

A. Physical exercise can slow the progression of type 2 diabetes mellitus. (Physical exercise slows the progression of type 2 diabetes mellitus because exercise has beneficial effects on carbohydrate metabolism and insulin sensitivity. Exercise improves blood glucose control in type 2 diabetes, reduces cardiovascular risk factors, contributes to weight loss, and improves well-being.)

During the lecture, the clinical instructor tells the students that 50% to 60% of daily calories should come from carbohydrates. What should the nurse say about the types of carbohydrates that can be eaten? A. Try to limit simple sugars to between 10% and 20% of daily calories. B. Simple carbohydrates are absorbed more rapidly than complex carbohydrates. C. Simple sugars cause a rapid spike in glucose levels and should be avoided. D. Simple sugars should never be consumed by someone with diabetes.

A. Try to limit simple sugars to between 10% and 20% of daily calories. (It is recommended that carbohydrates provide 50% to 60% of the daily calories. Approximately 40% to 50% should be from complex carbohydrates. The remaining 10% to 20 % of carbohydrates could be from simple sugars.)

Dr. Wijangco orders insulin lispro (Humalog) 10 units for Alicia, a client with diabetes mellitus. When will the nurse administer this medication? A. When the client is eating B. Thirty minutes before meals C. Fifteen minutes before meals D. When the meal trays arrive on the floor

A. When the client is eating (The onset action for the insulin lispro (Humalog) is 10 to 15 minutes so it must be given when the client is eating to prevent hypoglycemia. Insulin lispro is a new type of insulin. It starts working sooner than other insulin types. It also reaches peak activity faster and goes away sooner.)

45. Which nursing action can the nurse delegate to experienced unlicensed assistive personnel (UAP) who are working in the diabetic clinic? a. Measure the ankle-brachial index. b. Check for changes in skin pigmentation. c. Assess for unilateral or bilateral foot drop. d. Ask the patient about symptoms of depression.

ANS: A (Checking systolic pressure at the ankle and brachial areas and calculating the ankle-brachial index is a procedure that can be done by UAP who have been trained in the procedure. The other assessments require more education and critical thinking and should be done by the registered nurse (RN).)

34. A 27-yr-old patient admitted with diabetic ketoacidosis (DKA) has a serum glucose level of 732 mg/dL and serum potassium level of 3.1 mEq/L. Which action prescribed by the health care provider should the nurse take first? a. Place the patient on a cardiac monitor. b. Administer IV potassium supplements. c. Ask the patient about home insulin doses. d. Start an insulin infusion at 0.1 units/kg/hr.

ANS: A (Hypokalemia can lead to potentially fatal dysrhythmias such as ventricular tachycardia and ventricular fibrillation, which would be detected with electrocardiogram (ECG) monitoring. Because potassium must be infused over at least 1 hour, the nurse should initiate cardiac monitoring before infusion of potassium. Insulin should not be administered without cardiac monitoring because insulin infusion will further decrease potassium levels. Discussion of home insulin and possible causes can wait until the patient is stabilized.)

44. The nurse has administered 4 oz of orange juice to an alert patient whose blood glucose was 62 mg/dL. Fifteen minutes later, the blood glucose is 67 mg/dL. Which action should the nurse take next? a. Give the patient 4 to 6 oz more orange juice. b. Administer the PRN glucagon (Glucagon) 1 mg IM. c. Have the patient eat some peanut butter with crackers. d. Notify the health care provider about the hypoglycemia.

ANS: A (The "rule of 15" indicates that administration of quickly acting carbohydrates should be done two or three times for a conscious patient whose glucose remains less than 70 mg/dL before notifying the health care provider. More complex carbohydrates and fats may be used after the glucose has stabilized. Glucagon should be used if the patient's level of consciousness decreases so that oral carbohydrates can no longer be given.)

35. A patient with diabetic ketoacidosis is brought to the emergency department. Which prescribed action should the nurse implement first? a. Infuse 1 L of normal saline per hour. b. Give sodium bicarbonate 50 mEq IV push. c. Administer regular insulin 10 U by IV push. d. Start a regular insulin infusion at 0.1 units/kg/hr.

ANS: A (The most urgent patient problem is the hypovolemia associated with diabetic ketoacidosis (DKA), and the priority is to infuse IV fluids. The other actions can be done after the infusion of normal saline is initiated.)

47. After change-of-shift report, which patient should the nurse assess first? a. A 19-yr-old patient with type 1 diabetes who has a hemoglobin A1C of 12% b. A 23-yr-old patient with type 1 diabetes who has a blood glucose of 40 mg/dL c. A 50-yr-old patient who uses exenatide (Byetta) and is reporting acute abdominal pain d. A 40-yr-old patient who is pregnant and whose oral glucose tolerance test is 202 mg/dL

ANS: B (Because the brain requires glucose to function, untreated hypoglycemia can cause unconsciousness, seizures, and death. The nurse will rapidly assess and treat the patient with low blood glucose. The other patients also have symptoms that require assessments or interventions, but they are not at immediate risk for life-threatening complications.)

43. A few weeks after an 82-yr-old patient with a new diagnosis of type 2 diabetes has been placed on metformin (Glucophage) therapy, the home health nurse makes a visit. Which finding should the nurse promptly discuss with the health care provider? a. Hemoglobin A1C level is 7.9%. b. Glomerular filtration rate is decreased. c. Last eye examination was 18 months ago. d. Patient has questions about the prescribed diet.

ANS: B (The decrease in renal function may indicate a need to adjust the dose of metformin or change to a different medication. In older patients, the goal for A1C may be higher in order to avoid complications associated with hypoglycemia. The nurse will plan on scheduling the patient for an eye examination and addressing the questions about diet, but the area for prompt intervention is the patient's decreased renal function.)

38. Which laboratory value reported by the unlicensed assistive personnel (UAP) indicates an urgent need for the nurse to assess the patient? a. Bedtime glucose of 140 mg/dL b. Noon blood glucose of 52 mg/dL c. Fasting blood glucose of 130 mg/dL d. 2-hr postprandial glucose of 220 mg/dL

ANS: B (The nurse should assess the patient with a blood glucose level of 52 mg/dL for symptoms of hypoglycemia and give the patient a carbohydrate-containing beverage such as orange juice. The other values are within an acceptable range or not immediately dangerous for a patient with diabetes.)

28. A patient who has diabetes and reports burning foot pain at night receives a new prescription. Which information should the nurse teach the patient about amitriptyline? a. Amitriptyline decreases the depression caused by your foot pain. b. Amitriptyline helps prevent transmission of pain impulses to the brain. c. Amitriptyline corrects some of the blood vessel changes that cause pain. d. Amitriptyline improves sleep and makes you less aware of nighttime pain.

ANS: B (Tricyclic antidepressants (TCAs) decrease the transmission of pain impulses to the spinal cord and brain. TCAs also improve sleep quality and are used for depression, but that is not the major purpose for their use in diabetic neuropathy. TCAs do not affect the blood vessel changes that contribute to neuropathy.

42. Which patient statement indicates that the nurse's teaching about exenatide (Byetta) has been effective? a. "I may feel hungrier than usual when I take this medicine." b. "I will not need to worry about hypoglycemia with the Byetta." c. "I should take my daily aspirin at least an hour before the Byetta." d. "I will take the pill at the same time I eat breakfast in the morning."

ANS: C (Because exenatide slows gastric emptying, oral medications should be taken at least 1 hour before the exenatide to avoid slowing absorption. Exenatide is injected and increases feelings of satiety. Hypoglycemia can occur with this medication.)

39. When a patient with type 2 diabetes is admitted for a cholecystectomy, which nursing action can the nurse delegate to a licensed practical/vocational nurse (LPN/VN)? a. Communicate the blood glucose level and insulin dose to the circulating nurse in surgery. b. Discuss the reason for the use of insulin therapy during the immediate postoperative period. c. Administer the prescribed lispro (Humalog) insulin before transporting the patient to surgery. d. Plan strategies to minimize the risk for hypoglycemia or hyperglycemia during the postoperative period.

ANS: C (LPN/LVN education and scope of practice includes administration of insulin. Communication about patient status with other departments, planning, and patient teaching are skills that require RN education and scope of practice.)

36. A patient who was admitted with diabetic ketoacidosis secondary to a urinary tract infection has been weaned off an insulin drip 30 minutes ago. The patient reports feeling lightheaded and sweaty. Which action should the nurse take first? a. Infuse dextrose 50% by slow IV push. b. Administer 1 mg glucagon subcutaneously. c. Obtain a glucose reading using a finger stick. d. Have the patient drink 4 ounces of orange juice.

ANS: C (The patient's clinical manifestations are consistent with hypoglycemia, and the initial action should be to check the patient's glucose with a finger stick or order a stat blood glucose. If the glucose is low, the patient should ingest a rapid-acting carbohydrate, such as orange juice. Glucagon or dextrose 50% might be given if the patient's symptoms become worse or if the patient is unconscious.)

40. An active 32-yr-old male who has type 1 diabetes is being seen in the endocrine clinic. Which finding indicates a need for the nurse to discuss a possible a change in therapy with the health care provider? a. Hemoglobin A1C level of 6.2% b. Heart rate at rest of 58 beats/min c. Blood pressure of 140/88 mmHg d. High-density lipoprotein (HDL) level of 65 mg/dL

ANS: C (To decrease the incidence of macrovascular and microvascular problems in patients with diabetes, the blood pressure should be kept in normal range. An A1C less than 6.5%, a low resting heart rate (consistent with regular aerobic exercise in a young adult), and an HDL level of 65 mg/dL all indicate that the patient's diabetes and risk factors for vascular disease are well controlled.)

41. A 30-yr-old patient has a new diagnosis of type 2 diabetes. When should the nurse recommend the patient schedule a dilated eye examination? a. Every 2 years b. Every 6 months c. As soon as available d. At the age of 39 years

ANS: C Because many patients have some diabetic retinopathy when they are first diagnosed with type 2 diabetes, a dilated eye examination is recommended at the time of diagnosis and annually thereafter.

46. After change-of-shift report, which patient will the nurse assess first? a. A 19-yr-old patient with type 1 diabetes who was admitted with dawn phenomenon b. A 35-yr-old patient with type 1 diabetes whose most recent blood glucose reading was 230 mg/dL c. A 68-yr-old patient with type 2 diabetes who has severe peripheral neuropathy and reports burning foot pain d. A 60-yr-old patient with hyperosmolar hyperglycemic syndrome who has poor skin turgor and dry oral mucosa

ANS: D (The patient's diagnosis of HHS and signs of dehydration indicate that the nurse should rapidly assess for signs of shock and determine whether increased fluid infusion is needed. The other patients also need assessment and intervention but do not have life-threatening complications.)

A male client has just been diagnosed with type 1 diabetes mellitus. When teaching the client and family how diet and exercise affect insulin requirements, Nurse Joy should include which guideline? A. "You'll need more insulin when you exercise or increase your food intake." B. "You'll need less insulin when you exercise or reduce your food intake." C. "You'll need less insulin when you increase your food intake." D. "You'll need more insulin when you exercise or decrease your food intake."

B. "You'll need less insulin when you exercise or reduce your food intake."

7. A client was brought to the emergency room with complaints of slurring of speech, vomiting, dry mucosa, and dry skin turgor. Lab tests showing serum sodium 125 mEq/L and serum blood glucose of 350 mg/dL. Nurse Sophie will anticipate the physician to initially order which of the following intravenous solutions? A. 10% dextrose in water (D10W) B. 0.9% normal saline solution C. 5% dextrose in water (D5W) D. 0.45% normal saline solution

B. 0.9% normal saline solution (The client is experiencing diabetic ketoacidosis. Initial priority in the treatment is to restore the extracellular fluid volume through the intravenous administration of 0.9% normal saline at 15-20 ml/kg/h. Immediate fluid resuscitation is vital to correct hypovolemia, restore tissue perfusion, and to clear ketones. Hydration improves glycemic control independent of insulin.)

The goal of preprandial blood glucose for those with type 1 diabetes mellitus is: A. <80 mg/dl B. <130 mg/dl C. <180 mg/dl D. >8%

B. <130 mg/dl (According to the American Diabetes Association, the recommended preprandial glucose target for an adult with diabetes is between 80-130 mg/dl. In the management of diabetes, health care providers usually assess glycemic control with fasting plasma glucose (FPG) and premeal glucose measurements, as well as by measuring HbA1c.)

When taking a health history, the nurse screens for manifestations suggestive of diabetes type 1. Which of the following manifestations are considered the primary manifestations of type 1 diabetes mellitus and would be most suggestive and require follow-up investigation? A. Excessive intake of calories, rapid weight gain, and difficulty losing weight B. An increase in three areas: thirst, intake of fluids, and hunger C. Poor circulation, wound healing, and leg ulcers D. Lack of energy, weight g

B. An increase in three areas: thirst, intake of fluids, and hunger (The primary manifestations of diabetes type 1 are polyuria (increased urine output), polydipsia (increased thirst), polyphagia (increased hunger). Patients most commonly present hyperglycemic with polydipsia, polyuria, and polyphagia. Polyuria is secondary to osmotic diuresis, which is caused by hyperglycemia. Young children may present with nocturnal enuresis. Polydipsia is related to hyperosmolality and dehydration from increased urination.)

The nurse working in the physician's office is reviewing lab results on the clients seen that day. One of the clients who has classic diabetic symptoms had an eight-hour fasting plasma glucose (FPG) test done. The nurse realizes that diagnostic criteria developed by the American Diabetes Association for diabetes include classic diabetic symptoms plus which of the following fasting plasma glucose levels? A. Higher than 106 mg/dl B. Higher than 126 mg/dl C. Higher than 140 mg/dl D. Higher than

B. Higher than 126 mg/dl (Diabetes is diagnosed at a fasting blood glucose of greater than or equal to 126 mg/dl. Random venous blood glucose of at or above 11.1mmol/L (?200 mg/dL), or fasting blood glucose at or above 7 mmol/L (?126 mg/dL) on two or more separate occasions indicates the client is likely to have diabetes)

The nurse is admitting a client with newly diagnosed diabetes mellitus and left-sided heart failure. Assessment reveals low blood pressure, increased respiratory rate and depth, drowsiness, and confusion. The client complains of headache and nausea. Based on the serum laboratory results below, how would the nurse interpret the client's acid-base balance? Lab Results: pH: 7.34HCO3-: 19 mEq/LPaCO2: 35 mm HgPaO2: 88 mm HgPotassium: 5.3 mEq/LChloride: 102 mEq/LCalcium: 10.4 mg/dlAnion gap: 30 mEq/L A. Metabolic alkalosis. B. Metabolic acidosis. C. Respiratory acidosis. D. Respiratory alkalosis.

B. Metabolic acidosis. (This client has metabolic acidosis, which typically manifests with a low pH, low bicarbonate level, normal to low PaCO2 , and normal PaO2 . The client's serum electrolyte levels also support metabolic acidosis, which include an elevated potassium level, normal to elevated chloride level, and normal calcium level. The client's anion gap of 30 mEq/L is high, also indicative of metabolic acidosis. This kind of metabolic acidosis occurs with diabetic ketoacidosis and other disorders.)

Rosemary has been taking glargine (Lantus) to treat her condition. One of the benefits of glargine (Lantus) insulin is its ability to: A. Release insulin rapidly throughout the day to help control basal glucose. B. Release insulin evenly throughout the day and control basal glucose levels. C. Simplify the dosing and better control blood glucose levels during the day. D. Cause hypoglycemia with other manifestations of other adverse reactions.

B. Release insulin evenly throughout the day and control basal glucose levels. (Glargine (Lantus) insulin is designed to release insulin evenly throughout the day and control basal glucose levels. Insulin glargine is long-acting insulin injected once daily and provides a basal level of insulin throughout the day. Option A: Insulin glargine has an onset of action of 1.5 to 2 hours. It has a long duration of action up to 24 hours due to modifications of amino acids, including Asn to Gly at position 21 of the A chain and the addition of two Arg residues and positions 31 and 32 of the B chain. Option C: Regimens often combine it with rapid-acting insulin to obtain optimal glycemic control. Decisions of when to administer insulin glargine are dependent on the patient's schedule and the resultant effects that the insulin has on the particular patient's blood glucose levels. Option D: The most common adverse effect of insulin glargine is hypoglycemia. Special attention is necessary for insulin usage in fasting patients, those who have recently changed their diets or have had recent activity changes such as beginning a workout program.)

Harry is a diabetic patient who is experiencing a reaction to alternating periods of nocturnal hypoglycemia and hyperglycemia. The patient might be manifesting which of the following? A. Uncontrolled diabetes B. Somogyi phenomenon C. Brittle diabetes D. Diabetes insipidus

B. Somogyi phenomenon (The Somogyi phenomenon manifests itself with nocturnal hypoglycemia, followed by a marked increase in glucose and an increase in ketones. The Somogyi phenomenon states that early morning hyperglycemia occurs due to a rebound effect from late-night hypoglycemia.)

16. An ailing 70-year-old woman with a diagnosis of type 2 diabetes mellitus has been ill with pneumonia. The client's intake has been very poor, and she is admitted to the hospital for observation and management as needed. What is the most likely problem with this patient? A. Insulin resistance has developed. B. Diabetic ketoacidosis is occurring. C. Hypoglycemia unawareness is developing. D. Hyperglycemic hyperosmolar nonketotic syndrome.

D. Hyperglycemic hyperosmolar nonketotic syndrome. (Illness, especially with the frail elderly patient whose appetite is poor, can result in dehydration and HHNS. The most frequent reason for this complication is infection. The infectious process in the respiratory, gastrointestinal, and genitourinary systems can act as the causative factor. The reason for this is the insensible water loss and the release of endogenous catecholamines. Approximately 50% to 60% of HHS is attributable to infectious etiology. Option A: Insulin resistance usually is indicated by a daily insulin requirement of 200 units or more. Insulin resistance is primarily an acquired condition related to excess body fat, though genetic causes are identified as well. The clinical definition of insulin resistance remains elusive as there is not a generally accepted test for insulin resistance. Option B: Diabetic ketoacidosis, an acute metabolic condition, usually is caused by absent or markedly decreased amounts of insulin. Diabetic ketoacidosis (DKA) is characterized by hyperglycemia, acidosis, and ketonemia. It is a life-threatening complication of diabetes and is typically seen in patients with type-1 diabetes mellitus, though it may also occur in patients with type-2 diabetes mellitus. Option C: In patients who do not have diabetes, hypoglycemia is uncommon, but when it occurs, there are a few major causes of hypoglycemia: pharmacologic, alcohol, critical illness, counter-regulatory hormone deficiencies, and non-islet cell tumors.)

A nurse has a four-patient assignment in the medical step-down unit. When planning care for the clients, which client would have the following treatment goals: fluid replacement, vasopressin replacement, and correction of underlying intracranial pathology? A. The client with diabetes mellitus. B. The client with diabetes insipidus. C. The client with diabetic ketoacidosis. D. The client with syndrome of inappropriate antidiuretic hormone (SIADH) secretion.

B. The client with diabetes insipidus. (Maintaining adequate fluid, replacing vasopressin, and correcting underlying intracranial problems (typically lesions, tumors, or trauma affecting the hypothalamus or pituitary gland) are the main objectives in treating diabetes insipidus. Diabetes insipidus (DI) is a disease process that results in either decreased release of or response to antidiuretic hormone (ADH, also known as vasopressin or AVP), which can cause electrolyte imbalances.)

The guidelines for carbohydrate counting as medical nutrition therapy for diabetes mellitus includes all of the following, except: A. Flexibility in types and amounts of foods consumed B. Unlimited intake of total fat, saturated fat, and cholesterol C. Including adequate servings of fruits, vegetables, and the dairy group D. Applicable to with either Type 1 or Type 2 diabetes mellitus

B. Unlimited intake of total fat, saturated fat, and cholesterol (The guidelines for Carbohydrate Counting as medical nutrition therapy for diabetes mellitus include all of the following except unlimited intake of total fat, saturated fat, and cholesterol. Carb counting at its most basic level involves counting the number of grams of carbohydrate in a meal and matching that to the dose of insulin.)

somoygi effect treatment

Bedtime snack, reducing the dose of insulin or both

During a visit to the hospital, the student nurses are asked which of the following persons would most likely be diagnosed with diabetes mellitus. They are correct if they answered a 44-year-old: A. Caucasian woman. B. Asian woman. C. African-American woman. D. Hispanic male.

C. African-American woman. (Age-specific prevalence of diagnosed diabetes mellitus (DM) is higher for African-Americans and Hispanics than for Caucasians. Among those younger than 75, black women had the highest incidence. T2DM varies among ethnic groups and is 2 to 6 times more prevalent in Blacks, Native Americans, Pima Indians, and Hispanic Americans compared to Whites in the United States. Option A: Compared with Caucasians, racial minority populations are disproportionately affected by diabetes and have excessive risk for such complications as ESRD and amputations. The Multiple Risk Factor Intervention Trial (MRFIT) found that African Americans had a higher risk of diabetic ESRD than Caucasians. Option B: NHWs and Asian Americans have similar prevalence rates of 7.1% and 8.4%, respectively, where NHBs and Hispanic Americans overall have higher prevalence rates of 11.8% and 12.6%, respectively. Asian Indians have the highest diabetes prevalence rate (14.2%), whereas Asian Americans from Korea and Japan have the lowest diabetes prevalence rates 4.0% and 4.9%, respectively. Option D: In the United States, T1DM rates rose in most age and ethnic groups by about 2% yearly, and rates are higher in Hispanic youth. The exact reason for this pattern remains unknown. The incidence of T1DM has been increasing worldwide. In Europe, Australia, and the Middle East, rates are rising by 2% to 5% annually.)

Nurse Ronn is assessing a client with possible Cushing's syndrome. In a client with Cushing's syndrome, the nurse would expect to find: A. Hypotension. B. Thick, coarse skin. C. Deposits of adipose tissue in the trunk and dorsocervical area. D. Weight gain in arms and legs.

C. Deposits of adipose tissue in the trunk and dorsocervical area. (Because of changes in fat distribution, adipose tissue accumulates in the trunk, face (moonface), and dorsocervical areas (buffalo hump). Physical examination of the patient will reveal increased fat deposits in the upper half of the body leading to "Buffalo torso," characteristic moon facies (earlobes are not visible when viewed from the front), thin arms and legs, acne, hirsutism, proximal muscle weakness of shoulder and hip girdle muscles, paper-thin skin, abdominal pain due to gut perforation in rare cases, and wide vertical purplish abdominal striae.)

Joko has recently been diagnosed with type 1 Diabetes Mellitus and asks nurse Jessica for help formulating a nutrition plan. Which of the following recommendations would the nurse make to help the client increase calorie consumption to offset absorption problems? A. Eat small meals with two or three snacks throughout the day to keep blood glucose levels steady B. Increase the consumption of simple carbohydrates C. Eating small meals with two or three snacks may be more helpful in maintaining blood glucose levels than three large meals. D. Skip meals to help lose weight

C. Eating small meals with two or three snacks may be more helpful in maintaining blood glucose levels than three large meals. (Eating small meals with two or three snacks may be more helpful in maintaining blood glucose levels than three large meals. Complex carbohydrates (apples, broccoli, peas, dried beans, carrots, peas, oats) decrease glucose levels/insulin needs, reduce serum cholesterol levels, and promote satiation. Food intake is scheduled according to specific insulin characteristics and individual patient responses.)

Nurse Matt makes a home visit to the client with diabetes mellitus. During the visit, Nurse Matt notes the client's additional insulin vials are not refrigerated. What is the best action by the nurse at this time? A. Instruct the client to label each vial with the date when opened. B. Tell the client there is no need to keep additional vials. C. Have the client place the insulin vials in the refrigerator. D. Have the client discard the vials.

C. Have the client place the insulin vials in the refrigerator. (Vials not in use should be refrigerated to preserve drug potency. Vials of insulin not in use should be refrigerated. Extreme temperatures (<36 or >86°F, <2 or >30°C) and excess agitation should be avoided to prevent loss of potency, clumping, frosting, or precipitation.)

The nurse is working with an overweight client who has a high-stress job and smokes. This client has just received a diagnosis of type 2 diabetes mellitus and has just been started on an oral hypoglycemic agent. Which of the following goals for the client which if met, would be most likely to lead to an improvement in insulin efficiency to the point the client would no longer require oral hypoglycemic agents? A. Comply with medication regimen 100% for 6 months B. Quit the use of any tobacco pro

C. Lose a pound a week until the weight is within the normal range for height and exercise 30 minutes daily (When Type II diabetics lose weight through diet and exercise they sometimes have an improvement in insulin efficiency sufficient to the degree they no longer require oral hypoglycemic agents. A diet low in saturated fat, refined carbohydrates, high fructose corn syrup, and high in fiber and monounsaturated fats needs to be encouraged. Aerobic exercise for a duration of 90 to 150 minutes per week is also beneficial. The major target in T2DM patients, who are obese, is weight loss.)

Dr. Hugo has prescribed sulfonylureas for Rebecca in the management of diabetes mellitus type 2. As a nurse, you know that the primary purpose of sulfonylureas, such as long-acting glyburide (Micronase), is to: A. Induce hypoglycemia by decreasing insulin sensitivity. B. Improve insulin sensitivity and decrease hyperglycemia. C. Stimulate the beta cells of the pancreas to secrete insulin. D. Decrease insulin sensitivity by enhancing glucose uptake.

C. Stimulate the beta cells of the pancreas to secrete insulin. (Sulfonylureas such as glyburide are used only with patients who have some remaining pancreatic-beta cell function. These drugs stimulate insulin secretion, which reduces liver glucose output and increases cell uptake of glucose, enhancing the number of and sensitivity of cell receptor sites for interaction with insulin. Option A: Medications that reduce insulin resistance (insulin-sensitizing and antihyperglycemic effects) include metformin and thiazolidinediones. Metformin is a biguanide; it reduces hepatic glucose output and increases the uptake in the peripheral tissues (muscle and adipocytes). Option B: Thiazolidinediones (TZDs) are insulin sensitizers that act on intracellular metabolic pathways to enhance insulin action and increase insulin sensitivity in critical tissues. TZDs also increases adiponectin levels, decrease hepatic gluconeogenesis, and increase insulin-dependent glucose uptake in muscle and fat. Adiponectin, a cytokine secreted by fat tissue, increases insulin sensitivity, and fatty acid oxidation increases with TZD therapy. Option D: Metformin exerts its glucose-lowering effect by suppressing gluconeogenesis in the liver and facilitating glucose uptake and use by peripheral tissues. Decreased glucose uptake may result from suppressed insulin signaling or impaired glucose transporter (GLUT) 4 trafficking.)

Genevieve has diabetes type 1 and receives insulin for glycemic control. She tells the nurse that she likes to have a glass of wine with dinner. What will the best plan of the nurse for client education include? A. The alcohol could cause pancreatic disease. B. The alcohol could cause serious liver disease. C. The alcohol could predispose you to hypoglycemia. D. The alcohol could predispose you to hyperglycemia.

C. The alcohol could predispose you to hypoglycemia. (Alcohol can potentiate hypoglycemic, not hyperglycemic, effects in the client. When the client drinks alcohol, the alcohol can inhibit the liver's ability to release glucose into the blood. This can be particularly significant for people on stronger medication such as insulin because it can mean that the liver is not able to release enough glycogen to keep the blood glucose levels from going too low under the influence of insulin in the body.)

Desmopressin: Adverse effects: EENT

Conjunctivitis, epistaxis, laceration, nasal congestion (nasal form), ocular, edema, pharyngitis, rhinitis, sore throat

Desmopressin: Adverse effects: Resp

Cough, upper respiratory infections

Cushing's Syndrome or Addisons Disease: Moon face, Buffalo Hump

Cushing's syndrome

Gary has diabetes type 2. Nurse Martha has taught him about the illness and evaluates learning has occurred when the client makes which statement? A. "My cells have increased their receptors, but there is enough insulin." B. "My peripheral cells have increased sensitivity to insulin." C. "My beta cells cannot produce enough insulin for my cells." D. "My cells cannot use the insulin my pancreas makes."

D. "My cells cannot use the insulin my pancreas makes." (With type 2 diabetes mellitus, the pancreas produces insulin, but the cells cannot use it. T2DM is an insulin-resistance condition with associated beta-cell dysfunction. Initially, there is a compensatory increase in insulin secretion, which maintains glucose levels in the normal range. As the disease progresses, beta cells change, and insulin secretion is unable to maintain glucose homeostasis, producing hyperglycemia.

At the time Cherrie Ann found out that the symptoms of diabetes were caused by high levels of blood glucose, she decided to break the habit of eating carbohydrates. With this, the nurse would be aware that the client might develop which of the following complications? A. Retinopathy B. Atherosclerosis C. Glycosuria D. Acidosis

D. Acidosis (When a client's carbohydrate consumption is inadequate, ketones are produced from the breakdown of fat. These ketones lower the pH of the blood, potentially causing acidosis that can lead to a diabetic coma. Diabetic ketoacidosis (DKA) is characterized by hyperglycemia, acidosis, and ketonemia. It is a life-threatening complication of diabetes and typically seen in patients with type-1 diabetes mellitus, though it may also occur in patients with type-2 diabetes mellitus.)

The principal goals of therapy for older patients who have poor glycemic control are: A. Enhancing the quality of life. B. Decreasing the chance of complications. C. Improving self-care through education. The principal goals of therapy for older patients who have poor glycemic control are: A. Enhancing the quality of life. B. Decreasing the chance of complications. C. Improving self-care through education. D. All of the above.

D. All of the above.

During the morning rounds, Nurse AJ accompanied the physician in every patient's room. The physician writes orders for the client with diabetes mellitus. Which order would the nurse validate with the physician? A. Use Humalog insulin for sliding scale coverage. B. Metformin (Glucophage) 1000 mg per day in divided doses. C. Administer regular insulin 30 minutes prior to meals. D. Lantus insulin 20U BID.

D. Lantus insulin 20U BID. (Lantus insulin is usually prescribed once a day so an order for BID dosing should be validated with the physician. Lantus is designed to give a steady level of insulin over 24 hours, even when you are not eating such as between meals and overnight. This helps keep blood glucose levels consistent during the day and at night.)

A 50-year-old widower is admitted to the hospital with a diagnosis of diabetes mellitus and complaints of rapid-onset weight loss, elevated blood glucose levels, and polyphagia. The gerontology nurse should anticipate which of the following secondary medical diagnoses? A. Impaired glucose tolerance B. Gestational diabetes mellitus C. Pituitary tumor D. Pancreatic tumor

D. Pancreatic tumor (The onset of hyperglycemia in older adults can occur more slowly. When the older adult reports rapid-onset weight loss, elevated blood glucose levels, and polyphagia, the healthcare provider should consider pancreatic tumors. Weight loss occurs in about 90% of patients. Approximately 90% of all cases of pancreatic cancer are among people over 55 years of age. Option A: These patients are usually asymptomatic since they have prediabetes. The family history of diabetes and any previous history of gestational diabetes mellitus should be obtained. If they are diabetic, they will present with polyuria, polydipsia, infections, and neuropathy. Option B: The clinical features of gestational diabetes mellitus can be varied. The disproportionate weight gain, obesity, and elevated BMI can be suggestive features. The diagnosis is established by the laboratory screening method at the 24 to 28 weeks of pregnancy. Option C: Pituitary microadenoma is usually an incidental finding on MRI head. Patients are asymptomatic unless the tumor is hormonally active. Pituitary macroadenoma presents with mass effects and potentially hormonal deficiency or hormonal excess.)

An older woman with diabetes mellitus visits the clinic concerning her condition. Which of the following symptoms might an older woman with diabetes mellitus complain? A. Anorexia B. Pain intolerance C. Weight loss D. Perineal itching

D. Perineal itching (Older women might complain of perineal itching due to vaginal candidiasis. In diabetes, blood glucose levels can go abnormally high, which can therefore provide ideal conditions for naturally present yeast to grow and also diminishes the body's ability to fight infection.)

A male client with a tentative diagnosis of hyperosmolar hyperglycemic nonketotic syndrome (HHNS) has a history of type 2 diabetes that is being controlled with an oral diabetic agent, tolazamide (Tolinase). Which of the following is the most important laboratory test for confirming this disorder? A. Serum potassium level B. Serum sodium level C. Arterial blood gas (ABG) values D. Serum osmolarity

D. Serum osmolarity (Serum osmolarity is the most important test for confirming HHNS; it's also used to guide treatment strategies and determine evaluation criteria. A client with HHNS typically has a serum osmolarity of more than 350 mOsm/L. The serum osmolality is determined by the formula 2Na + Glucose /18 + BUN / 2.8. The resultant hyperglycemia increases the serum osmolarity to a significant degree. The glucose level in HHS is usually above 600 mg/dL. Hyperglycemia also creates an increase in the osmotic gradient with free water drawn out from the extravascular space from the increased osmotic gradient.)

Desmopressin 3 D's

Diabetes Insididus, Decrease Urine Output, Death by headache (Seizures)

Most common disorder of the Posterior Lobe; deficiency in anti diuretic hormone (ADH)

Diabetes Insipidus

Pheochromocytoma fight or flight

Facial flushing, palpitations Increased BP,HR Glucose Increased Headaches Tremors Frequent Sweating Loss of weight Increased anxiety and fear Growing tumor Heat intolerance Tired and weak

12. A patient receives aspart (NovoLog) insulin at 8:00 AM. At which time would the nurse anticipate the highest risk for hypoglycemia? a. 10:00 AM b. 12:00 AM c. 2:00 PM d. 4:00 PM

a. 10:00 AM (The rapid-acting insulins peak in 1 to 3 hours. The patient is not at a high risk for hypoglycemia at the other listed times, although hypoglycemia may occur.)

Desmopressin: Adverse effects: Skin

Flushing

Diabetes FIT

Frequency- 3 to 4X a week Intensity 60%-80% of Maximal Heart Rate Time Aerobic 20-30 minutes/5-10 minute warm up

Pheochromocytoma 5H's

HTN (250/150) Headache Hyperhidrosis (sweat) Hyper metabolism Hyperglycemia

Most common cause of hypothyroidism

Hashimoto's thyroiditis

Thyroidectomy Post Diet

High Calorie Diet 4000-5000 cal Increased protein 1-2 g/kg Increased carb (6meals)

Desmopressin: Adverse effects: Cardiovascular

Hypertension (high doses), myocardial infarction, thrombosis, transient hypotension

Myxedema coma complications

Impaired consciousness- coma Subnormal temperature, hypotension, hyperventilation Cardiovascular collapse: hypoglycemia, hyponatremia, and lactic acidosis

Dawn phenomenon treatment

Increase in an insulin or an adjustment in administration time

Addison's Disease Etiology & Pathophysiology

Primary: Lack of glucocorticoids, mineralocorticoids, and androgens Secondary: Lack of pituitary ACTH (Adrenocorticotropic hormone), Lack of glucocorticoids and androgens

How do you store desmopressin nasal version?

Refrigerate

Triiodothyronine (T3)

Secreted by the thyroid gland; T3 increases metabolism in cells. (20% of hormone, More potent than T4

Thyroxine (T4)

Secreted by the thyroid gland; also called tetraiodothyronine. T4 increases metabolism in cells. (90% of hormone)

Hypernatremia S/S

Seizures* Muscle weakness Restlessness Extreme thirst Confusion Lethargy Irritability Unconsciousness

Endocrine activity controlled by specific mechanisms that either

Stimulate or inhibit, hormone synthesis and secretion

Glypizide

Sulfonylruea: increase insulin release from pancreas (major side effect hypoglycemia)

thyroid gland produces

T3, T4, calcitonin

Dawn phenomenon

The dawn phenomenon is a natural rise in blood sugar that occurs in the early morning hours. Symptoms are similar to hyperglycemia, and may include nausea, fatigue, and extreme thirst.

pituitary gland

The endocrine system's most influential gland. Under the influence of the hypothalamus, the pituitary regulates growth and controls other endocrine glands.

HHS treatment

Very similar to DKA; Fluids, more volume than DKA, delivered more slowly

Desmopressin: Adverse effects: GU

Vulvar pain (Parenteral form)

pheochromocytoma

a benign tumor of the adrenal medulla that causes the gland to produce excess epinephrine

Clients with hypermagnesemia exhibit

a loss of deep tendon reflexes, coma, or cardiac arrest. The most frequent symptoms and signs may include weakness, nausea, dizziness, and confusion (less than 7.0 mg/dL). Increasing values (7 to 12 mg/dL) induce decreased reflexes, worsening confusional state, drowsiness, bladder paralysis, flushing, headache, and constipation. A slight reduction in blood pressure and blurred vision caused by diminished accommodation and convergence can manifest.

25. Which question during the assessment of a patient who has diabetes will help the nurse identify autonomic neuropathy? a. "Do you feel bloated after eating?" b. "Have you seen any skin changes?" c. "Do you need to increase your insulin dosage when you are stressed?" d. "Have you noticed any painful new ulcerations or sores on your feet?"

a. "Do you feel bloated after eating?" (Autonomic neuropathy can cause delayed gastric emptying, which results in a bloated feeling for the patient. The other questions are also appropriate to ask but would not help in identifying autonomic neuropathy.)

20. The nurse is preparing to teach a 43-yr-old man who is newly diagnosed with type 2 diabetes about home management of the disease. Which action should the nurse take first? a. Assess the patient's perception of what it means to have diabetes. b. Ask the patient's family to participate in the diabetes education program. c. Demonstrate how to check glucose using capillary blood glucose monitoring. d. Discuss the need for the patient to actively participate in diabetes management.

a. Assess the patient's perception of what it means to have diabetes. (Before planning teaching, the nurse should assess the patient's interest in and ability to self-manage the diabetes. After assessing the patient, the other nursing actions may be appropriate, but planning needs to be specific to each patient.)

26. Which information will the nurse include in teaching a female patient who has peripheral arterial disease, type 2 diabetes, and sensory neuropathy of the feet and legs? a. Choose flat-soled leather shoes. b. Set heating pads on a low temperature. c. Use a callus remover for corns or calluses. d. Soak feet in warm water for an hour each day.

a. Choose flat-soled leather shoes. (The patient is taught to avoid high heels and that leather shoes are preferred. The feet should be washed, but not soaked, in warm water daily. Heating pad use should be avoided. Commercial callus and corn removers should be avoided. The patient should see a specialist to treat these problems.)

9. Which nursing action is most important in assisting an older patient who has diabetes to engage in moderate daily exercise? a. Determine what types of activities the patient enjoys. b. Remind the patient that exercise improves self-esteem. c. Teach the patient about the effects of exercise on glucose level. d. Give the patient a list of activities that are moderate in intensity.

a. Determine what types of activities the patient enjoys. (Because consistency with exercise is important, assessment for the types of exercise that the patient finds enjoyable is the most important action by the nurse in ensuring adherence to an exercise program. The other actions may be helpful but are not the most important in improving compliance.)

20. Which finding for a patient who has hypothyroidism and hypertension indicates that the nurse should contact the health care provider before administering levothyroxine (Synthroid)? a. Increased thyroxine (T4) level b. Blood pressure 112/62 mm Hg c. Distant and difficult to hear heart sounds d. Elevated thyroid stimulating hormone level

a. Increased thyroxine (T4) level (An increased thyroxine level indicates the levothyroxine dose needs to be decreased. The other data are consistent with hypothyroidism and the nurse should administer the levothyroxine.)

23. Which finding indicates to the nurse that the current therapies are effective for a patient who has acute adrenal insufficiency? a. Increasing serum sodium levels b. Decreasing blood glucose levels c. Decreasing serum chloride levels d. Increasing serum potassium levels

a. Increasing serum sodium levels (Clinical manifestations of Addison's disease include hyponatremia and an increase in sodium level indicates improvement. The other values indicate that treatment has not been effective.)

14. A patient with diabetes is starting on intensive insulin therapy. Which type of insulin will the nurse discuss using for mealtime coverage? a. Lispro (Humalog) b. Glargine (Lantus) c. Detemir (Levemir) d. NPH (Humulin N)

a. Lispro (Humalog) (Rapid- or short-acting insulin is used for mealtime coverage for patients receiving intensive insulin therapy. NPH, glargine, or detemir will be used as the basal insulin.)

42. Which finding by the nurse when assessing a patient with Hashimoto's thyroiditis and a goiter will require the most immediate action? a. New-onset changes in the patient's voice b. Elevation in the patient's T3 and T4 levels c. Resting apical pulse rate 112 beats/min d. Bruit audible bilaterally over the thyroid gland

a. New-onset changes in the patient's voice (Changes in the patient's voice indicate that the goiter is compressing the laryngeal nerve and may lead to airway compression. The other findings will also be reported but are expected with Hashimoto's thyroiditis and do not require immediate action.)

32. The nurse is caring for a patient admitted with diabetes insipidus (DI). Which information is most important to report to the health care provider? a. The patient is confused and lethargic. b. The patient reports a recent head injury. c. The patient has a urine output of 400 mL/hr. d. The patient's urine specific gravity is 1.003.

a. The patient is confused and lethargic. (The patient's confusion and lethargy may indicate hypernatremia and should be addressed quickly. In addition, patients with DI compensate for fluid losses by drinking copious amounts of fluids, but a patient who is lethargic will be unable to drink enough fluids and will become hypovolemic. A high urine output, low urine specific gravity, and history of a recent head injury are consistent with diabetes insipidus, but they do not require immediate nursing action to avoid life-threatening complications.)

13. Which patient action indicates an accurate understanding of the nurse's teaching about the use of an insulin pump? a. The patient programs the pump for an insulin bolus after eating. b. The patient changes the location of the insertion site every week. c. The patient takes the pump off at bedtime and starts it again each morning. d. The patient plans a diet with more calories than usual when using the pump.

a. The patient programs the pump for an insulin bolus after eating. (In addition to the basal rate of insulin infusion, the patient will adjust the pump to administer a bolus after each meal, with the dosage depending on the oral intake. The insertion site should be changed every 2 or 3 days. There is more flexibility in diet and exercise when an insulin pump is used, but it does not provide for consuming a higher calorie diet. The pump will deliver a basal insulin rate 24 hours a day.)

6. The nurse is assessing a 55-yr-old female patient with type 2 diabetes who has a body mass index (BMI) of 31 kg/m2 .Which goal in the plan of care is most important for this patient? a. The patient will reach a glycosylated hemoglobin level of less than 7%. b. The patient will follow a diet and exercise plan that results in weight loss. c. The patient will choose a diet that distributes calories throughout the day. d. The patient will state the reasons for eliminating simple sugars in the diet.

a. The patient will reach a glycosylated hemoglobin level of less than 7%. (The complications of diabetes are related to elevated blood glucose and the most important patient outcome is the reduction of glucose to near-normal levels. A BMI of 30.9/kg/m2 or above is considered obese, so the other outcomes are appropriate but are not as high in priority.)

1. A 40-year-old patient with suspected acromegaly is seen at the clinic. To assist in making the diagnosis, which question should the nurse ask? a. "Have you had a recent head injury?" b. "Do you have to wear larger shoes now?" c. "Is there a family history of acromegaly?" d. "Are you experiencing tremors or anxiety?"

b. "Do you have to wear larger shoes now?" (Acromegaly causes an enlargement of the hands and feet. Head injury and family history are not risk factors for acromegaly. Tremors and anxiety are not clinical manifestations of acromegaly.)

8. Which statement by the patient who has newly diagnosed type 1 diabetes indicates a need for additional instruction from the nurse? a. "I will need a bedtime snack because I take an evening dose of NPH insulin." b. "I can choose any foods, as long as I use enough insulin to cover the calories." c. "I can have an occasional beverage with alcohol if I include it in my meal plan." d. "I will eat something at meal times to prevent hypoglycemia, even if I am not hungry."

b. "I can choose any foods, as long as I use enough insulin to cover the calories." (Most patients with type 1 diabetes need to plan diet choices very carefully. Patients who are using intensified insulin therapy have considerable flexibility in diet choices but still should restrict dietary intake of items such as fat, protein, and alcohol. The other patient statements are correct and indicate good understanding of the diet instruction.)

24. The nurse admits a patient to the hospital in Addisonian crisis. Which patient statement supports the need to plan additional teaching? a. "I frequently eat at restaurants, and my food has a lot of added salt." b. "I had the flu earlier this week, so I couldn't take the hydrocortisone." c. "I always double my dose of hydrocortisone on the days that I go for a long run." d. "I take twice as much hydrocortisone in the morning dose as I do in the afternoon."

b. "I had the flu earlier this week, so I couldn't take the hydrocortisone." (The need for hydrocortisone replacement is increased with stressors such as illness, and the patient needs to be taught to call the health care provider because medication and IV fluids and electrolytes may need to be given. The other patient statements indicate appropriate management of the Addison's disease.)

10. Which patient statement to the nurse indicates a need for additional instruction in administering insulin? a. "I can buy the 0.5-mL syringes because the line markings are easier to see." b. "I need to rotate injection sites among my arms, legs, and abdomen each day." c. "I do not need to aspirate the plunger to check for blood before injecting insulin." d. "I should draw up the regular insulin first, after injecting air into the NPH bottle."

b. "I need to rotate injection sites among my arms, legs, and abdomen each day." (Rotating sites is no longer recommended because there is more consistent insulin absorption when the same site is used consistently. The other patient statements are accurate and indicate that no additional instruction is needed.)

25. A 29-year-old woman with systemic lupus erythematosus has been prescribed 2 weeks of high-dose prednisone therapy. Which information about the prednisone is most important for the nurse to include? a. "Weigh yourself daily to monitor for weight gain." b. "The prednisone dose should be decreased gradually." c. "A weight-bearing exercise program will help minimize risk for osteoporosis." d. "Call the health care provider if you have mood changes with the prednisone."

b. "The prednisone dose should be decreased gradually." (Acute adrenal insufficiency may occur if exogenous corticosteroids are suddenly stopped. Mood alterations and weight gain are possible adverse effects of corticosteroid use, but these are not life-threatening effects. Osteoporosis occurs when patients take corticosteroids for longer periods.)

40. Which question will the nurse in the endocrine clinic ask to help determine a patient's risk factors for goiter? a. "How much milk do you drink?" b. "What medications are you taking?" c. "Have you had a recent neck injury?" d. "Are your immunizations up to date?"

b. "What medications are you taking?" Medications that contain thyroid-inhibiting substances can cause goiter. Milk intake, neck injury, and immunization history are not risk factors for goiter.

39. After receiving change-of-shift report about the following four patients, which patient should the nurse assess first? a. A 31-year-old female patient with Cushing syndrome and a blood glucose level of 244 mg/dL b. A 70-year-old female patient taking levothyroxine (Synthroid) who has an irregular pulse of 134 c. A 53-year-old male patient who has Addison's disease and is due for a prescribed dose of hydrocortisone (Solu-Cortef). d. A 22-year-old male patient admitted with syndrome of inappropriate antidiuretic hormone (SIADH) who has a serum sodium level of 130 mEq/L

b. A 70-year-old female patient taking levothyroxine (Synthroid) who has an irregular pulse of 134 (Initiation of thyroid replacement in older adults may cause angina and cardiac dysrhythmias. The patient's high pulse rate needs rapid investigation by the nurse to assess for and intervene with any cardiac problems. The other patients also require nursing assessment and/or actions but are not at risk for life-threatening complications.)

11. A patient who had a subtotal thyroidectomy earlier today develops laryngeal stridor and a cramp in the right hand upon returning to the surgical nursing unit. Which collaborative action will the nurse anticipate next? a. Plan for emergency tracheostomy. b. Administer IV calcium gluconate. c. Prepare for endotracheal intubation. d. Begin thyroid hormone replacement.

b. Administer IV calcium gluconate. (The patient's clinical manifestations of stridor and cramping are consistent with tetany caused by hypocalcemia resulting from damage to the parathyroid glands during surgery. Endotracheal intubation or tracheostomy may be needed if the calcium does not resolve the stridor. Thyroid hormone replacement may be needed eventually but will not improve the symptoms of hypocalcemia.)

37. A patient is admitted with diabetes insipidus. Which action will be appropriate for the registered nurse (RN) to delegate to an experienced licensed practical/vocational nurse (LPN/VN)? a. Titrate the infusion of 5% dextrose in water. b. Administer prescribed subcutaneous DDAVP. c. Assess the patient's overall hydration status every 8 hours. d. Teach the patient to use desmopressin (DDAVP) nasal spray.

b. Administer prescribed subcutaneous DDAVP. (Administration of medications is included in LPN/VN education and scope of practice. Assessments, patient teaching, and titrating fluid infusions are more complex skills and should be done by the RN.)

14. Which nursing assessment of a 70-year-old patient is most important to make during initiation of thyroid replacement with levothyroxine (Synthroid)? a. Fluid balance b. Apical pulse rate c. Nutritional intake d. Orientation and alertness

b. Apical pulse rate (In older patients, initiation of levothyroxine therapy can increase myocardial oxygen demand and cause angina or dysrhythmias. The medication also is expected to improve mental status and fluid balance and will increase metabolic rate and nutritional needs, but these changes will not result in potentially life-threatening complications.)

19. A patient who had radical neck surgery to remove a malignant tumor developed hypoparathyroidism. What topic should the nurse plan to teach the patient? a. Bisphosphonates to reduce bone demineralization b. Calcium supplements to normalize serum calcium levels c. Increasing fluid intake to decrease risk for nephrolithiasis d. Including whole grains in the diet to prevent constipation

b. Calcium supplements to normalize serum calcium levels (Oral calcium supplements are used to maintain the serum calcium in normal range and prevent the complications of hypocalcemia. Whole grain foods decrease calcium absorption and will not be recommended. Bisphosphonates will lower serum calcium levels further by preventing calcium from being reabsorbed from bone. Kidney stones are not a complication of hypoparathyroidism and low calcium levels.)

7. A patient who has type 1 diabetes plans to swim laps for an hour daily at 1:00 PM. What advice should the clinic nurse plan to give the patient? a. Increase the morning dose of NPH insulin (Novolin N). b. Check glucose level before, during, and after swimming. c. Time the morning insulin injection to peak while swimming. d. Delay eating the noon meal until after finishing the swimming.

b. Check glucose level before, during, and after swimming. (The change in exercise will affect blood glucose, and the patient will need to monitor glucose carefully to determine the need for changes in diet and insulin administration. Because exercise tends to decrease blood glucose, patients are advised to eat before exercising. Increasing the morning NPH or timing the insulin to peak during exercise may lead to hypoglycemia, especially with the increased exercise.)

19. Which action by the patient who is self-monitoring blood glucose indicates a need for additional teaching? a. Washes the puncture site using warm water and soap. b. Chooses a puncture site in the center of the finger pad. c. Hangs the arm down for a minute before puncturing the site. d. Says the result of 120 mg indicates good blood sugar control.

b. Chooses a puncture site in the center of the finger pad. (The patient is taught to choose a puncture site at the side of the finger pad because there are fewer nerve endings along the side of the finger pad. The other patient actions indicate that teaching has been effective.)

4. The nurse is assessing a male patient diagnosed with a pituitary tumor causing panhypopituitarism. Which assessment finding is consistent with panhypopituitarism? a. High blood pressure b. Decreased facial hair c. Elevated blood glucose d. Intermittent tachycardia

b. Decreased facial hair (Changes in male secondary sex characteristics such as decreased facial hair, testicular atrophy, diminished spermatogenesis, loss of libido, impotence, and decreased muscle mass are associated with decreases in follicle-stimulating hormone (FSH) and luteinizing hormone (LH). Fasting hypoglycemia and hypotension occur in panhypopituitarism because of decreases in adrenocorticotropic hormone (ACTH) and cortisol. Bradycardia is due to the decrease in thyroid-stimulating hormone (TSH) and thyroid hormones associated with panhypopituitarism.)

8. A 56-year-old patient who is disoriented and reports a headache and muscle cramps is hospitalized with syndrome of inappropriate antidiuretic hormone (SIADH). Which initial laboratory result should the nurse expect? a. Elevated hematocrit b. Decreased serum sodium c. Increased serum chloride d. Low urine specific gravity

b. Decreased serum sodium (When water is retained, the serum sodium level will drop below normal, causing the clinical manifestations reported by the patient. The hematocrit will decrease because of the dilution caused by water retention. Urine will be more concentrated with a higher specific gravity. The serum chloride level will usually decrease along with the sodium level.)

12. Which nursing action will be included in the plan of care for a patient with Graves' disease who has exophthalmos? a. Place cold packs on the eyes to relieve pain and swelling. b. Elevate the head of the patient's bed to reduce periorbital fluid. c. Apply alternating eye patches to protect the corneas from irritation. d. Teach the patient to blink every few seconds to lubricate the corneas.

b. Elevate the head of the patient's bed to reduce periorbital fluid. (The patient should sit upright as much as possible to promote fluid drainage from the periorbital area. With exophthalmos, the patient is unable to close the eyes completely to blink. Lubrication of the eyes, rather than eye patches, will protect the eyes from developing corneal scarring. The swelling of the eye is not caused by excessive blood flow to the eye, so cold packs will not be helpful.)

17. A patient with primary hyperparathyroidism has a serum phosphorus level of 1.7 mg/dL (0.55 mmol/L) and calcium of 14 mg/dL (3.5 mmol/L). Which nursing action should be included in the plan of care? a. Restrict the patient to bed rest. b. Encourage 4000 mL of fluids daily. c. Institute routine seizure precautions. d. Assess for positive Chvostek's sign.

b. Encourage 4000 mL of fluids daily. (The patient with hypercalcemia is at risk for kidney stones, which may be prevented by a high fluid intake. Seizure precautions and monitoring for Chvostek's or Trousseau's sign are appropriate for hypocalcemic patients. The patient should engage in weight-bearing exercise to decrease calcium loss from bone.)

24. Which action should the nurse take after a patient treated with intramuscular glucagon for hypoglycemia regains consciousness? a. Assess the patient for symptoms of hyperglycemia. b. Give the patient a snack of peanut butter and crackers. c. Have the patient drink a glass of orange juice or nonfat milk. d. Administer a continuous infusion of 5% dextrose for 24 hours.

b. Give the patient a snack of peanut butter and crackers. (Rebound hypoglycemia can occur after glucagon administration, but having a meal containing complex carbohydrates plus protein and fat will help prevent hypoglycemia. Orange juice and nonfat milk will elevate blood glucose rapidly, but the cheese and crackers will stabilize blood glucose. Administration of IV glucose might be used in patients who were unable to take in nutrition orally. The patient should be assessed for symptoms of hypoglycemia after glucagon administration.)

15. Which information will the nurse include when teaching a patient who has type 2 diabetes about glyburide? a. Glyburide decreases glucagon secretion from the pancreas. b. Glyburide stimulates insulin production and release from the pancreas. c. Glyburide should be taken even if the morning blood glucose level is low. d. Glyburide should not be used for 48 hours after receiving IV contrast media.

b. Glyburide stimulates insulin production and release from the pancreas. (The sulfonylureas stimulate the production and release of insulin from the pancreas. If the glucose level is low, the patient should contact the health care provider before taking glyburide because hypoglycemia can occur with this class of medication. Metformin should be held for 48 hours after administration of IV contrast media, but this is not necessary for glyburide. Glyburide does not affect glucagon secretion.)

5. A patient with type 2 diabetes is scheduled for a follow-up visit in the clinic several months from now. Which test will the nurse schedule to evaluate the effectiveness of treatment for the patient? a. Fasting blood glucose b. Glycosylated hemoglobin c. Oral glucose tolerance test d. Urine dipstick for glucose and ketones

b. Glycosylated hemoglobin (The glycosylated hemoglobin (A1C) test shows the overall control of glucose over 90 to 120 days. A fasting blood level indicates only the glucose level at one time. Urine glucose testing is not an accurate reflection of blood glucose level and does not reflect the glucose over a prolonged time. Oral glucose tolerance testing is done to diagnose diabetes but is not used for monitoring glucose control after diabetes has been diagnosed.)

27. What finding should the nurse plan to assess for in a patient diagnosed with a pheochromocytoma? a. Flushing b. Headache c. Bradycardia d. Hypoglycemia

b. Headache (The classic clinical manifestations of pheochromocytoma are hypertension, tachycardia, severe headache, diaphoresis, and abdominal or chest pain. Elevated blood glucose may also occur because of sympathetic nervous system stimulation. Bradycardia and flushing would not be expected.)

35. A patient has just arrived in the postanesthesia recovery unit (PACU) after a thyroidectomy. Which information about the patient is most important to communicate to the surgeon? a. Difficult to awaken. b. Increasing neck swelling. c. Reports 7/10 incisional pain. d. Cardiac rate 112 beats/min.

b. Increasing neck swelling. (The neck swelling may lead to respiratory difficulty, and rapid intervention is needed to prevent airway obstruction. The incisional pain should be treated but is not unusual after surgery. A heart rate of 112 beats/min is not unusual in a patient who has been hyperthyroid and has just arrived in the PACU from surgery. Sleepiness in the immediate postoperative period is expected.)

5. Which information will the nurse include when teaching a 50-year-old male patient about somatropin (Genotropin)? a. The medication will be needed for 3 to 6 months. b. Inject the medication subcutaneously every day. c. Blood glucose levels may decrease when taking the medication. d. Stop taking the medication if swelling of the hands or feet occurs.

b. Inject the medication subcutaneously every day. (Somatropin is injected subcutaneously daily, preferably in the evening. The patient will need to continue on somatropin for life. If swelling or other common adverse effects occur, the health care provider should be notified. Growth hormone will increase blood glucose levels.)

21. An unresponsive patient who has type 2 diabetes is brought to the emergency department and diagnosed with hyperosmolar hyperglycemia syndrome (HHS). What should the nurse anticipate doing? a. Giving 50% dextrose b. Inserting an IV catheter c. Initiating O2 by nasal cannula d. Administering glargine (Lantus) insulin

b. Inserting an IV catheter (HHS is initially treated with large volumes of IV fluids to correct hypovolemia. Regular insulin is administered, not a long-acting insulin. There is no indication that the patient requires O2. Dextrose solutions will increase the patient's blood glucose and would be contraindicated.)

3. The nurse is planning postoperative care for a patient who is being admitted to the surgical unit from the recovery room after transsphenoidal resection of a pituitary tumor. Which nursing action should be included? a. Palpate extremities for edema. b. Measure urine volume every hour. c. Check hematocrit every 2 hours for 8 hours. d. Monitor continuous pulse oximetry for 24 hours.

b. Measure urine volume every hour. (After pituitary surgery, the patient is at risk for diabetes insipidus caused by cerebral edema. Monitoring of urine output and urine specific gravity is essential. Hemorrhage is not a common problem. There is no need to check the hematocrit hourly. The patient is at risk for dehydration, not volume overload. The patient is not at high risk for problems with oxygenation, and continuous pulse oximetry is not needed.)

26. What action should the nurse take when providing care for a patient who has an adrenocortical adenoma causing hyperaldosteronism? a. Check blood glucose level every 4 hours. b. Monitor the blood pressure every 4 hours. c. Elevate the patient's legs to relieve edema. d. Order the patient a potassium-restricted diet.

b. Monitor the blood pressure every 4 hours. (Hypertension caused by sodium retention is a common complication of hyperaldosteronism. Hyperaldosteronism does not cause an elevation in blood glucose. The patient will be hypokalemic and require potassium supplementation before surgery. Edema does not usually occur with hyperaldosteronism.)

44. The cardiac telemetry unit charge nurse receives status reports from other nursing units about four patients who need cardiac monitoring. Which patient should be transferred to the cardiac unit first? a. Patient with Hashimoto's thyroiditis and a heart rate of 102 b. Patient with tetany who has a new order for IV calcium chloride c. Patient with Cushing syndrome and a blood glucose of 140 mg/dL d. Patient with Addison's disease who takes IV hydrocortisone twice daily

b. Patient with tetany who has a new order for IV calcium chloride (Emergency treatment of tetany requires IV administration of calcium; electrocardiographic monitoring will be required because cardiac arrest may occur if high calcium levels result from too-rapid administration. The information about the other patients indicates that they are more stable than the patient with tetany.)

16. A patient who was admitted with myxedema coma and diagnosed with hypothyroidism is improving. Discharge is expected to occur in 2 days. Which teaching strategy is likely to result in effective patient self-management at home? a. Delay teaching until closer to discharge date. b. Provide written reminders of information taught. c. Offer multiple options for management of therapies. d. Ensure privacy for teaching by asking the family to leave.

b. Provide written reminders of information taught. (Written instructions will be helpful to the patient because initially the hypothyroid patient may be unable to remember to take medications and other aspects of self-care. Because the treatment regimen is complex, teaching should be started well before discharge. Family members or friends should be included in teaching because the hypothyroid patient is likely to forget some aspects of the treatment plan. A simpler regimen will be easier to understand until the patient is euthyroid.)

33. The nurse is taking a health history from a 29-yr-old patient at the first prenatal visit. The patient reports that she has no personal history of diabetes, but her mother has diabetes. Which action will the nurse plan to take? a. Teach the patient about administering regular insulin. b. Schedule the patient for a fasting blood glucose level. c. Teach about an increased risk for fetal problems with gestational diabetes. d. Schedule an oral glucose tolerance test for the twenty-fourth week of pregnancy.

b. Schedule the patient for a fasting blood glucose level. Patients at high risk for gestational diabetes should be screened for diabetes on the initial prenatal visit. An oral glucose tolerance test may also be used to check for diabetes, but it would be done before the twenty-fourth week. Teaching plans would depend on the outcome of a fasting blood glucose test and other tests.

11. Which patient action indicates accurate understanding of the nurse's teaching about administration of aspart (NovoLog) insulin? a. The patient avoids injecting the insulin into the upper abdominal area. b. The patient cleans the skin with soap and water before insulin administration. c. The patient stores the insulin in the freezer after administering the prescribed dose. d. The patient pushes the plunger down while removing the syringe from the injection site.

b. The patient cleans the skin with soap and water before insulin administration. (Cleaning the skin with soap and water is acceptable. Insulin should not be frozen. The patient should leave the syringe in place for about 5 seconds after injection to be sure that all the insulin has been injected. The upper abdominal area is one of the preferred areas for insulin injection.)

6. Which finding indicates to the nurse that demeclocycline is effective for a patient with syndrome of inappropriate antidiuretic hormone (SIADH)? a. Weight has increased. b. Urinary output is increased. c. Peripheral edema is increased. d. Urine specific gravity is increased.

b. Urinary output is increased. (Demeclocycline blocks the action of antidiuretic hormone (ADH) on the renal tubules and increases urine output. An increase in weight or an increase in urine specific gravity indicates that the SIADH is not corrected. Peripheral edema does not occur with SIADH. A sudden weight gain without edema is a common clinical manifestation of this disorder.)

nocturnal emuresis

bed-wetting during sleep

4. The nurse is assessing a 22-yr-old patient experiencing the onset of symptoms of type 1 diabetes. To which question would the nurse anticipate a positive response? a. "Are you anorexic?" b. "Is your urine dark colored?" c. "Have you lost weight lately?" d. "Do you crave sugary drinks?"

c. "Have you lost weight lately?" (Weight loss occurs because the body is no longer able to absorb glucose and starts to break down protein and fat for energy. The patient is thirsty but does not necessarily crave sugar-containing fluids. Increased appetite is a classic symptom of type 1 diabetes. With the classic symptom of polyuria, urine will be very dilute.)

31. A patient with diabetes rides a bicycle to and from work every day. Which site should the nurse teach the patient to use to administer the morning insulin? a. Thigh b. Buttock c. Abdomen d. Upper arm

c. Abdomen (Patients should be taught not to administer insulin into a site that will be exercised because exercise will increase the rate of absorption. The thigh, buttock, and arm are all exercised by riding a bicycle.)

10. Which information will the nurse teach a patient who has been newly diagnosed with Graves' disease? a. Exercise is contraindicated to avoid increasing metabolic rate. b. Restriction of iodine intake is needed to reduce thyroid activity. c. Antithyroid medications may take several months for full effect. d. Surgery will eventually be required to remove the thyroid gland.

c. Antithyroid medications may take several months for full effect. (Medications used to block the synthesis of thyroid hormones may take 2 to 3 months before the full effect is seen. Large doses of iodine are used to inhibit the synthesis of thyroid hormones. Exercise using large muscle groups is encouraged to decrease the irritability and hyperactivity associated with high levels of thyroid hormones. Radioactive iodine is the most common treatment for Graves' disease, although surgery may be used.)

30. A patient has just arrived on the unit after a thyroidectomy. Which action should the nurse take first? a. Observe the dressing for bleeding. b. Check the blood pressure and pulse. c. Assess the patient's respiratory effort. d. Support the patient's head with pillows.

c. Assess the patient's respiratory effort. Airway obstruction is a possible complication after thyroidectomy because of swelling or bleeding at the site or tetany. The priority nursing action is to assess the airway. The other actions are also part of the standard nursing care post thyroidectomy, but are not as high of a priority.

31. The nurse is caring for a patient following an adrenalectomy. What is the highest priority in the immediate postoperative period? a. Protecting the patient's skin b. Monitoring for signs of infection c. Balancing fluids and electrolytes d. Preventing emotional disturbances

c. Balancing fluids and electrolytes (After adrenalectomy, the patient is at risk for circulatory instability caused by fluctuating hormone levels, and the focus of care is to assess and maintain fluid and electrolyte status through the use of IV fluids and corticosteroids. The other goals are also important for the patient but are not as immediately life threatening as the circulatory collapse that can occur with fluid and electrolyte disturbances.)

30. Which action by a patient indicates that the home health nurse's teaching about glargine and regular insulin has been successful? a . The patient administers the glargine 30 minutes before each meal. b. The patient's family prefills the syringes with the mix of insulins weekly. c. The patient discards the open vials of glargine and regular insulin after 4 weeks. d. The patient draws up the regular insulin and then the glargine in the same syringe.

c. The patient discards the open vials of glargine and regular insulin after 4 weeks. (Insulin can be stored at room temperature for 4 weeks. Glargine should not be mixed with other insulins or prefilled and stored. Short-acting regular insulin is administered before meals, and glargine is given once daily.)

1. Which statement by a nurse to a patient newly diagnosed with type 2 diabetes is accurate? a. Insulin is not used to control blood glucose in patients with type 2 diabetes. b. Complications of type 2 diabetes are less serious than those of type 1 diabetes. c. Changes in diet and exercise may control blood glucose levels in type 2 diabetes. d. Type 2 diabetes is usually diagnosed when a patient is admitted in hyperglycemic coma.

c. Changes in diet and exercise may control blood glucose levels in type 2 diabetes. (For some patients with type 2 diabetes, changes in lifestyle are sufficient to achieve blood glucose control. Insulin is frequently used for type 2 diabetes, complications are equally severe as for type 1 diabetes, and type 2 diabetes is usually diagnosed with routine laboratory testing or after a patient develops complications such as frequent yeast infections.)

23. The health care provider suspects the Somogyi effect in a 50-yr-old patient whose 6:00 AM blood glucose is 230 mg/dL. Which action will the nurse teach the patient to take? a. Avoid snacking right before bedtime. b. Increase the rapid-acting insulin dose. c. Check the blood glucose during the night. d. Administer a larger dose of long-acting insulin.

c. Check the blood glucose during the night. (If the Somogyi effect is causing the patient's increased morning glucose level, the patient will experience hypoglycemia between 2:00 and 4:00 AM. The dose of insulin will be reduced, rather than increased. A bedtime snack is used to prevent hypoglycemic episodes during the night.)

15. An 82-year-old patient in a long-term care facility is newly diagnosed with hypothyroidism. The nurse will need to consult with the health care provider before administering the prescribed a. Docusate (Colace) b. Ibuprofen (Motrin) c. Diazepam (Valium) d. Cefoxitin (Mefoxin)

c. Diazepam (Valium) (Worsening of mental status and myxedema coma can be precipitated using sedatives, especially in older adults. The nurse should discuss the use of diazepam with the health care provider before administration. The other medications may be given safely to the patient.)

9. Which problem should the nurse anticipate for a patient admitted to the hospital with diabetes insipidus? a. Generalized edema b. Fluid volume overload c. Disturbed sleep pattern d. Decreased gas exchange

c. Disturbed sleep pattern (Nocturia occurs because of the polyuria caused by diabetes insipidus. Edema, excess fluid volume, and fluid retention are not expected.)

18. A patient develops carpopedal spasms and tingling of the lips following a parathyroidectomy. Which action will provide the patient with rapid temporary relief from the symptoms? a. Start the PRN O2 at 2 L/min per cannula. b. Administer the prescribed muscle relaxant. c. Have the patient rebreathe from a paper bag. d. Stretch the muscles with passive range of motion.

c. Have the patient rebreathe from a paper bag. (The patient's symptoms suggest mild hypocalcemia. The symptoms of hypocalcemia will be temporarily reduced by having the patient breathe into a paper bag, which will raise the PaCO2 and create a more acidic pH. Applying as-needed O2 or range of motion will have no impact on the ionized calcium level. Calcium supplements will be given to normalize calcium levels quickly, but oral supplements will take time to be absorbed.)

2. A patient screened for diabetes at a clinic has a fasting plasma glucose level of 120 mg/dL (6.7 mmol/L). What should the nurse plan to teach the patient? a. Self-monitoring of blood glucose b. Using low doses of regular insulin c. Lifestyle changes to lower blood glucose d. Effects of oral hypoglycemic medications

c. Lifestyle changes to lower blood glucose (The patient's impaired fasting glucose indicates prediabetes, and the patient should be counseled about lifestyle changes to prevent the development of type 2 diabetes. The patient with prediabetes does not require insulin or oral hypoglycemics for glucose control and does not need to self-monitor blood glucose.)

22. A 26-yr-old female who has type 1 diabetes develops a sore throat and runny nose after caring for her sick toddler. The patient calls the clinic for advice about her symptoms and reports a blood glucose level of 210 mg/dL despite taking her usual glargine (Lantus) and lispro (Humalog) insulin. What should the nurse advise the patient to do? a. Use only the lispro insulin until the symptoms are resolved. b. Limit intake of calories until the glucose is less than 120 mg/dL. c. Monitor blood glucose every 4 hours and contact the clinic if it rises. d. Decrease carbohydrates until glycosylated hemoglobin is less than 7%.

c. Monitor blood glucose every 4 hours and contact the clinic if it rises. (Infection and other stressors increase blood glucose levels and the patient will need to test blood glucose frequently, treat elevations appropriately with lispro insulin, and call the health care provider if glucose levels continue to be elevated. Discontinuing the glargine will contribute to hyperglycemia and may lead to diabetic ketoacidosis (DKA). Decreasing carbohydrate or caloric intake is not appropriate because the patient will need more calories when ill. Glycosylated hemoglobin testing is not used to evaluate short-term alterations in blood glucose.)

29. Which intervention will the nurse include in the plan of care for a patient with syndrome of inappropriate antidiuretic hormone (SIADH)? a. Encourage fluids to 2 to 3 L/day. b. Monitor for increasing peripheral edema. c. Offer the patient hard candies to suck on. d. Keep head of bed elevated to 30 degrees.

c. Offer the patient hard candies to suck on. (Sucking on hard candies decreases thirst for a patient on fluid restriction. Patients with SIADH are on fluid restrictions of 800 to 1000 mL/day. Peripheral edema is not seen with SIADH. The head of the bed is elevated no more than 10 degrees to increase left atrial filling pressure and decrease antidiuretic hormone (ADH) release.)

28. What topic should the nurse teach a patient who had a pituitary adenoma after the hypophysectomy? a. Sodium restriction to prevent fluid retention b. Insulin to maintain normal blood glucose levels c. Oral corticosteroids to replace endogenous cortisol d. Chemotherapy to prevent malignant tumor recurrence

c. Oral corticosteroids to replace endogenous cortisol (Antidiuretic hormone (ADH), cortisol, and thyroid hormone replacement will be needed for life after hypophysectomy. Without the effects of adrenocorticotropic hormone (ACTH) and cortisol, the blood glucose and serum sodium will be low unless cortisol is replaced. An adenoma is a benign tumor, and chemotherapy will not be needed.)

43. Which information obtained by the nurse in the endocrine clinic about a patient who has been taking prednisone 40 mg daily for 3 weeks is most important to report to the health care provider? a. Patient's blood pressure is 148/94 mm Hg. b. Patient has bilateral 2+ pitting ankle edema. c. Patient stopped taking the medication 2 days ago. d. Patient has not been taking the prescribed vitamin D.

c. Patient stopped taking the medication 2 days ago. (Sudden cessation of corticosteroids after taking the medication for a week or more can lead to adrenal insufficiency, with problems such as severe hypotension and hypoglycemia. The patient will need immediate evaluation by the health care provider to prevent or treat adrenal insufficiency. The other information will also be reported but does not require rapid treatment.)

21. A patient is being admitted with a diagnosis of Cushing syndrome. Which finding will the nurse expect during the assessment? a. Chronically low blood pressure b. Bronzed appearance of the skin c. Purplish streaks on the abdomen d. Decreased axillary and pubic hair

c. Purplish streaks on the abdomen (Purplish-red striae on the abdomen are a common clinical manifestation of Cushing syndrome. Hypotension and bronzed-appearing skin are manifestations of Addison's disease. Decreased axillary and pubic hair occur with androgen deficiency.)

18. A hospitalized patient who is diabetic received 38 U of NPH insulin at 7:00 AM. At 1:00 PM, the patient has been away from the nursing unit for 2 hours, missing the lunch delivery while awaiting a chest x-ray. What is the best action by the nurse to prevent hypoglycemia? a. Plan to discontinue the evening dose of insulin. b. Save the lunch tray for the patient's later return. c. Request that if testing is further delayed, the patient will eat lunch first. d. Send a glass of orange juice to the patient in the diagnostic testing area.

c. Request that if testing is further delayed, the patient will eat lunch first. (Consistency for mealtimes assists with regulation of blood glucose, so the best option is for the patient to have lunch at the usual time. Waiting to eat until after the procedure is likely to cause hypoglycemia. Holding the insulin dose later will not prevent hypoglycemia form the peak of the NPH dose. A glass of juice will keep the patient from becoming hypoglycemic but will cause a rapid rise in blood glucose because of the rapid absorption of the simple carbohydrate in these items.)

13. A patient who has hyperthyroidism is treated with radioactive iodine (RAI). What information should the nurse include in discharge teaching? a. Take radioactive precautions with all body secretions. b. Symptoms of hyperthyroidism should be relieved in about a week. c. Symptoms of hypothyroidism will occur as the RAI therapy takes effect. d. Discontinue the antithyroid medications that were taken before the RAI therapy.

c. Symptoms of hypothyroidism will occur as the RAI therapy takes effect. (There is a high incidence of post radiation hypothyroidism after RAI, and the patient should be monitored for symptoms of hypothyroidism. RAI has a delayed response, with the maximum effect not seen for 2 to 3 months, and the patient will continue to take antithyroid medications during this time. The therapeutic dose of radioactive iodine is low enough that no radiation safety precautions are needed.)

34. Which assessment finding for an adult admitted with Graves' disease requires the most rapid intervention by the nurse? a. Heart rate 136 beats/min b. Severe bilateral exophthalmos c. Temperature 103.8° F (40.4° C) d. Blood pressure 166/100 mm Hg

c. Temperature 103.8° F (40.4° C) (The patient's temperature indicates that the patient may have thyrotoxic crisis and that interventions to lower the temperature are needed immediately. The other findings also require intervention but do not indicate potentially life-threatening complications.)

17. When a patient who takes metformin (Glucophage) to manage type 2 diabetes develops an allergic rash from an unknown cause, the health care provider prescribes prednisone. What should the nurse anticipate? a. The patient may need a diet higher in calories while receiving prednisone. b. The patient may develop acute hypoglycemia while taking the prednisone. c. The patient may require administration of insulin while taking prednisone. d. The patient may have rashes caused by metformin-prednisone interactions.

c. The patient may require administration of insulin while taking prednisone. (Glucose levels increase when patients are taking corticosteroids, and insulin may be required to control blood glucose. Hypoglycemia is not a side effect of prednisone. Rashes are not an adverse effect caused by taking metformin and prednisone simultaneously. The patient may have an increased appetite when taking prednisone but will not need a diet that is higher in calories.)

29. A patient who has type 2 diabetes is being prepared for an elective coronary angiogram. Which information would the nurse anticipate might lead to rescheduling the test? a. The patient's most recent A1C was 7.5%. b. The patient's blood glucose is 128 mg/dL. c. The patient took the prescribed metformin today. d. The patient took the prescribed enalapril 4 hours ago.

c. The patient took the prescribed metformin today. (To avoid lactic acidosis, metformin should be discontinued a day or 2 before the coronary angiogram and should not be used for 48 hours after IV contrast media are administered. The other patient data will be reported but do not indicate any need to reschedule the procedure.)

somoygi effect diagnosis

check for hypoglycemia in the middle of the night leading to hyperglycemia in the morning

16. The nurse has been teaching a patient with type 2 diabetes about managing blood glucose levels and taking glipizide (Glucotrol). Which patient statement indicates a need for additional teaching? a. "If I overeat at a meal, I will still take the usual dose of medication." b. "Other medications besides the Glucotrol may affect my blood sugar." c. "When I am ill, I may have to take insulin to control my blood sugar." d. "My diabetes won't cause complications because I don't need insulin."

d. "My diabetes won't cause complications because I don't need insulin." (The patient should understand that type 2 diabetes places the patient at risk for many complications and that good glucose control is as important when taking oral agents as when using insulin. The other statements are accurate and indicate good understanding of the use of glipizide.)

41. Which finding by the nurse when assessing a patient with a large pituitary adenoma is most important to report to the health care provider? a. Changes in visual field b. Milk leaking from breasts c. Blood glucose 150 mg/dL d. Nausea and projectile vomiting

d. Nausea and projectile vomiting (Nausea and projectile vomiting may indicate increased intracranial pressure, which will require rapid actions for diagnosis and treatment. Changes in the visual field, elevated blood glucose, and galactorrhea are common with pituitary adenoma, but these do not require rapid action to prevent life-threatening complications.)

33. Which prescribed medication should the nurse expect will have rapid effects on a patient admitted to the emergency department in thyroid storm? a. Iodine b. Methimazole c. Propylthiouracil d. Propranolol (Inderal)

d. Propranolol (Inderal) (B-Adrenergic blockers work rapidly to decrease the cardiovascular manifestations of thyroid storm. The other medications take days to weeks to have an impact on thyroid function.)

22. A 44-year-old female patient with Cushing syndrome is admitted for adrenalectomy. Which intervention by the nurse will be helpful for the patient problem of disturbed body image related to changes in appearance? a. Reassure the patient that the physical changes are very common in patients with Cushing syndrome. b. Discuss the use of diet and exercise in controlling the weight gain associated with Cushing syndrome. c. Teach the patient that the metabolic impact of Cushing syndrome is of more importance than appearance. d. Remind the patient that most of the physical changes caused by Cushing syndrome will resolve after surgery.

d. Remind the patient that most of the physical changes caused by Cushing syndrome will resolve after surgery. (The most reassuring and accurate communication to the patient is that the physical and emotional changes caused by the Cushing syndrome will resolve after hormone levels return to normal postoperatively. Reassurance that the physical changes are expected or that there are more serious physiologic problems associated with Cushing syndrome minimize the patient's concerns. The patient's physiological changes are caused by the high hormone levels, not by the patient's diet or exercise choices.)

36. Which assessment finding in a patient who had a bilateral adrenalectomy requires the most rapid action by the nurse? a. The blood glucose is 192 mg/dL. b. The lungs have bibasilar crackles. c. The patient reports 6/10 incisional pain. d. The blood pressure (BP) is 88/50 mm Hg.

d. The blood pressure (BP) is 88/50 mm Hg. (The decreased BP indicates possible adrenal insufficiency. The nurse should immediately notify the health care provider so that corticosteroid medications can be administered. The nurse should also address the elevated glucose, incisional pain, and crackles with appropriate collaborative or nursing actions, but prevention and treatment of acute adrenal insufficiency are the priorities after adrenalectomy.)

38. Which information is most important for the nurse to communicate rapidly to the health care provider about a patient admitted with possible syndrome of inappropriate antidiuretic hormone (SIADH)? a. The patient has a recent weight gain of 9 pounds. b. The patient complains of dyspnea with activity. c. The patient has a urine specific gravity of 1.025. d. The patient has a serum sodium level of 118 mEq/L.

d. The patient has a serum sodium level of 118 mEq/L. (A serum sodium of less than 120 mEq/L increases the risk for complications such as seizures and needs rapid correction. The other data are not unusual for a patient with SIADH and do not indicate the need for rapid action.)

32. The nurse is interviewing a new patient with diabetes who takes rosiglitazone (Avandia). Which information would the nurse anticipate resulting in the health care provider discontinuing the medication? a. The patient's blood pressure is 154/92. b. The patient's blood glucose is 86 mg/dL. c. The patient reports a history of emphysema. d. The patient has chest pressure when walking.

d. The patient has chest pressure when walking. (Rosiglitazone can cause myocardial ischemia. The nurse should immediately notify the health care provider and expect orders to discontinue the medication. A blood glucose level of 86 mg/dL indicates a positive effect from the medication. Hypertension and a history of emphysema do not contraindicate this medication.)

3. A 28-yr-old male patient with type 1 diabetes reports how he manages his exercise and glucose control. Which behavior indicates that the nurse should implement additional teaching? a. The patient always carries hard candies when engaging in exercise. b. The patient goes for a vigorous walk when his glucose is 200 mg/dL. c. The patient has a peanut butter sandwich before going for a bicycle ride. d. The patient increases daily exercise when ketones are present in the urine.

d. The patient increases daily exercise when ketones are present in the urine. (When the patient is ketotic, exercise may result in an increase in blood glucose level. Patients with type 1 diabetes should be taught to avoid exercise when ketosis is present. The other statements are correct.)

27. Which finding indicates a need to contact the health care provider before the nurse administers metformin (Glucophage)? a. The patient's blood glucose level is 174 mg/dL. b. The patient is scheduled for a chest x-ray in an hour. c. The patient has gained 2 lb (0.9 kg) in the past 24 hours. d. The patient's estimated glomerular filtration rate is 42 mL/min.

d. The patient's estimated glomerular filtration rate is 42 mL/min. (The glomerular filtration rate indicates possible renal impairment, and metformin should not be used in patients with significant renal impairment. The other findings are not contraindications to the use of metformin.)

Alpha Cells produce

glucagon which increases blood glucose levels

Somogyi effect

happens when a low blood sugar (hypoglycemia) episode overnight leads to high blood sugar (hyperglycemia) in the morning due to a surge of hormones.

insulin hypertrophy

increased swelling of fat at the site of repeated injections

Insulin lipodystrophy

loss of subcutaneous fat and appears as slight dimpling or more serious pitting of subcutaneous fat; the use of human insulin helps prevent this complication

Afrezza

rapid acting inhaled insulin. It is given at the beginning of each meal or within 20 minutes after the meal. Must be used in combination with long acting insulin in patients with Type 1 Diabetes. (contraindication not to treat DKA, Side effect bronchospasm so patients with Asthma or COPD or smokers should not use this medication. Other side effects include hypoglycemia, cough, and throat pain or aggitation)

glucose tolerance test (GTT)

test for initial diagnosis of diabetes mellitus; patient is given dose of glucose; then blood samples are taken at regular intervals to determine patient's ability to use glucose properly (for gestational diabetes pregnancy)

Positive feedback

Increasing hormone levels, cause another gland to release a hormone that stimulates further release of the first hormone. It means to stop the release of the first hormone is required or it's release will continue.

Which instruction about insulin administration should nurse Kate give to a client? A. "Always follow the same order when drawing the different insulins into the syringe." B. "Shake the vials before withdrawing the insulin." C. "Store unopened vials of insulin in the freezer at temperatures well below freezing." D. "Discard the intermediate-acting insulin if it appears cloudy."

A. "Always follow the same order when drawing the different insulins into the syringe." (The client should be instructed always to follow the same order when drawing the different insulins into the syringe. Insulin is considered the most potent anabolic hormone known until today, and its effects on the body are necessary for tissue development, growth, and maintenance of glucose homeostasis. Insulin action starts by binding to two cell receptors, which are alpha and beta, that are linked by two disulfide bonds into a complex that is a heterotetrameric membrane.)

Dr. Kennedy prescribes glipizide (Glucotrol), an oral antidiabetic agent, for a male client with type 2 diabetes mellitus who has been having trouble controlling the blood glucose level through diet and exercise. Which medication instruction should the nurse provide? A. "Be sure to take glipizide 30 minutes before meals." B. "Glipizide may cause a low serum sodium level, so make sure you have your sodium level checked monthly." C. "You won't need to check your blood glucose level after you start taking glipizide." D. "Take glipizide after a meal to prevent heartburn."

A. "Be sure to take glipizide 30 minutes before meals." (The client should take glipizide twice a day, 30 minutes before a meal, because food decreases its absorption. The immediate release dosage form should be administered 30 minutes before meals to achieve the most significant reduction in postprandial hyperglycemia. Administration of the extended-release dosage form should be with breakfast or the first meal of the day. Practitioners should instruct patients to swallow the tablets whole and not to chew, split, or crush the tablets.)

A female client with Cushing's syndrome is admitted to the medical-surgical unit. During the admission assessment, nurse Tyzz notes that the client is agitated and irritable, has poor memory, reports loss of appetite, and appears disheveled. These findings are consistent with which problem? A. Depression B. Neuropathy C. Hypoglycemia D. Hyperthyroidism

A. Depression (Agitation, irritability, poor memory, loss of appetite, and neglect of one's appearance may signal depression, which is common in clients with Cushing's syndrome. In some studies, as many as 90% of Cushing's patients suffer from depression. In part, this is due to actual chemical changes in the brain from high cortisol. The depressing effect of having a serious and impairing illness may also contribute to depression.)

Nurse Wayne is aware that a positive Chvostek's sign indicates? A. Hypocalcemia B. Hyponatremia C. Hypokalemia D. Hypermagnesemia

A. Hypocalcemia (Chvostek's sign is elicited by tapping the client's face lightly over the facial nerve, just below the temple. If the client's facial muscles twitch, it indicates hypocalcemia. Chvostek's sign is another manifestation of heightened neuromuscular excitability. It is the spasm of facial muscles in response to tapping the facial nerve near the angle of the jaw.)

For the first 72 hours after thyroidectomy surgery, nurse Jamie would assess the female client for Chvostek's sign and Trousseau's sign because they indicate which of the following? A. Hypocalcemia B. Hypercalcemia C. Hypokalemia D. Hyperkalemia

A. Hypocalcemia (The client who has undergone a thyroidectomy is at risk for developing hypocalcemia from inadvertent removal or damage to the parathyroid gland. The client with hypocalcemia will exhibit a positive Chvostek's sign (facial muscle contraction when the facial nerve in front of the ear is tapped) and a positive Trousseau's sign (carpal spasm when a blood pressure cuff is inflated for a few minutes). These signs aren't present with hypercalcemia, hypokalemia, or hyperkalemia. Option B: Severe hypercalcemia inhibits neuromuscular and myocardial depolarization leading to muscle weakness and arrhythmias. Cardiovascular effects include prolonged PR interval, short QT interval, widened QRS complex, and bradycardia. Increased thirst with polydipsia and polyuria is seen initially, progressing to nephrolithiasis and nephrocalcinosis in chronic cases. Option C: Significant muscle weakness occurs at serum potassium levels below 2.5 mmol/L but can occur at higher levels if the onset is acute. Similar to the weakness associated with hyperkalemia, the pattern is ascending in nature affecting the lower extremities, progressing to involve the trunk and upper extremities and potentially advancing to paralysis. Option D: Physical exam findings may include hypertension and edema in the setting or renal disease. There may also be signs of hypoperfusion. Muscle tenderness may be present in patients with rhabdomyolysis. Jaundice may be seen in patients with hemolytic conditions. Patients may have muscle weakness, flaccid paralysis, or depressed deep tendon reflexes.)

Nurse Noemi administers glucagon to her diabetic client, then monitors the client for adverse drug reactions and interactions. Which type of drug interacts adversely with glucagon? A. Oral anticoagulants B. Anabolic steroids C. Beta-adrenergic blockers D. Thiazide diuretics

A. Oral anticoagulants (As a normal body protein, glucagon only interacts adversely with oral anticoagulants, increasing the anticoagulant effects. It doesn't interact adversely with anabolic steroids, beta-adrenergic blockers, or thiazide diuretics. Glucagon is a medication used in the management and treatment of hypoglycemia, as an antidote to beta-blocker and calcium channel blocker overdose, anaphylaxis refractory to epinephrine, and to aid in passing food boluses. It is in the anti-hypoglycemic class of medications.)

In a 29-year-old female client who is being successfully treated for Cushing's syndrome, nurse Lyzette would expect a decline in: A. Serum glucose level B. Hair loss C. Bone mineralization D. Menstrual flow

A. Serum glucose level (Hyperglycemia, which develops from glucocorticoid excess, is a manifestation of Cushing's syndrome. With successful treatment of the disorder, serum glucose levels decline. Cortisol is a steroid hormone, and it directly affects the transcription and translation of enzyme proteins involved in the metabolism of fats, glycogen, proteins synthesis, and Kreb's cycle. It promotes the production of free glucose in the body, elevating glucose levels, while simultaneously increasing insulin resistance.)

Nurse Ruth is assessing a client after a thyroidectomy. The assessment reveals muscle twitching and tingling, along with numbness in the fingers, toes, and mouth area. The nurse should suspect which complication? A. Tetany B. Hemorrhage C. Thyroid storm D. Laryngeal nerve damage

A. Tetany (Tetany may result if the parathyroid glands are excised or damaged during thyroid surgery. Hypoparathyroidism results in hypocalcemia which may become symptomatic and life-threatening. Criteria for hyperparathyroidism have not been clearly established, but the reported incidence is approximately 1/3 with the majority of these being temporary. It is important to maintain a consistent protocol for calcium management after total or completion thyroidectomy to minimize related complications.)

Desmopressin: Adverse effects: GI

Abdominal cramps, nausea

Octreotide (Sandostatin) indications

Acromegaly, Diarrhea, Dumping syndrome

Desmopressin: Adverse effects: other

Anaphylaxis, hyponatremia, injection, site, pain, and redness, water intoxication

Desmopressin: Adverse effects: CNS

Asthenia, chills, CVA, dizziness, headache (Serious signs of Hyponatremia: AMS and Seizures)

A female client whose physical findings suggest a hyperpituitary condition undergoes an extensive diagnostic workup. Test results reveal a pituitary tumor, which necessitates a transsphenoidal hypophysectomy. The evening before the surgery, nurse Jacob reviews preoperative and postoperative instructions given to the client earlier. Which postoperative instruction should the nurse emphasize? A. "You must lie flat for 24 hours after surgery." B. "You must avoid coughing, sneezing, and blowing your nose." C. "You must restrict your fluid intake." D. "You must report ringing in your ears immediately."

B. "You must avoid coughing, sneezing, and blowing your nose." (After a transsphenoidal hypophysectomy, the client must refrain from coughing, sneezing, and blowing the nose for several days to avoid disturbing the surgical graft used to close the wound. The need for nasal packs is dependent on the type of reconstructive technique and the surgeon's choice (used only in a minority of cases). The nasal pack is removed on postoperative day 1. Septal splints are warranted in traditional sublabial-transseptal-transsphenoidal approaches and removed on a postoperative day 5 to 7.)

Which of these signs suggests that a male client with the syndrome of inappropriate antidiuretic hormone (SIADH) secretion is experiencing complications? A. Tetanic contractions B. Neck vein distention C. Weight loss D. Polyuria

B. Neck vein distention (SIADH secretion causes antidiuretic hormone overproduction, which leads to fluid retention. Severe SIADH can cause such complications as vascular fluid overload, signaled by neck vein distention. Physical examination should include assessment of volume status, as these patients are typically euvolemic. Skin turgor and blood pressure are within the normal range. Moist mucous membranes with no evidence of jugular venous pulsation or edema typically indicate euvolemia.)

Nurse Troy is aware that the most appropriate nursing diagnosis for a patient with Addison's disease is which of the following? A. Excessive fluid volume B. Risk for infection C. Urinary retention D. Hypothermia

B. Risk for infection (Addison's disease decreases the production of all adrenal hormones, compromising the body's normal stress response and increasing the risk of infection. Other appropriate nursing diagnoses for a client with Addison's disease include Deficient fluid volume and Hyperthermia.)

A male client with primary diabetes insipidus is ready for discharge on desmopressin (DDAVP). Which instruction should nurse Lina provide? A. "Administer desmopressin while the suspension is cold." B. "Your condition isn't chronic, so you won't need to wear a medical identification bracelet." C. "You may not be able to use desmopressin nasally if you have nasal discharge or blockage." D. "You won't need to monitor your fluid intake and output after you start taking desmopressin."

C. "You may not be able to use desmopressin nasally if you have nasal discharge or blockage." (Desmopressin may not be absorbed if the intranasal route is compromised. Desmopressin administration can be utilized to distinguish between central vs. nephrogenic diabetes insipidus, with a positive response noted in central diabetes insipidus, meaning the kidneys respond appropriately to desmopressin with the expected concentration of the urine and increased reabsorption of fluids, resulting in eutonic urine.)

Nurse Perry is caring for a female client with type 1 diabetes mellitus who exhibits confusion, light-headedness, and aberrant behavior. The client is still conscious. The nurse should first administer: A. I.M. or subcutaneous glucagon. B. I.V. bolus of dextrose 50%. C. 15 to 20 g of a fast-acting carbohydrate such as orange juice. D. 10 U of fast-acting insulin.

C. 15 to 20 g of a fast-acting carbohydrate such as orange juice. (This client is having a hypoglycemic episode. Because the client is conscious, the nurse should first administer a fast-acting carbohydrate, such as orange juice, hard candy, or honey. For conscious patients able to take oral (PO) medications, readily absorbable carbohydrate sources (such as fruit juice) should be given. If the client has lost consciousness, the nurse should administer either I.M. or subcutaneous glucagon or an I.V. bolus of dextrose 50%. Option A: For patients unable to take oral agents, a 1-mg intramuscular (IM) injection of glucagon can be administered. Once the patient is more awake, a complex carbohydrate food source should be given to the patient to achieve sustained euglycemia. More frequent blood glucose monitoring should occur to rule out further drops in blood sugar. Option B: Identification of a hypoglycemic patient is critical due to potential adverse effects, including coma and/or death. Severe hypoglycemia can be treated with intravenous (IV) dextrose followed by infusion of glucose. Patients should be advised to wear a medical alert bracelet and to carry a glucose source like gel, candy, or tablets on their person in case symptoms arise. Option D: The nurse shouldn't administer insulin to a client who's hypoglycemic; this action will further compromise the client's condition. Nonpharmacological management of recurrent hypoglycemia involves patient education and lifestyle changes. Some patients are unaware of the serious ramifications of persistent hypoglycemia. As such, patients should be educated on the importance of routine blood glucose monitoring as well as on the identification of the individual's symptoms of hypoglycemia.)

Nurse Perry is caring for a female client with type 1 diabetes mellitus who exhibits confusion, light-headedness, and aberrant behavior. The client is still conscious. The nurse should first administer: A. I.M. or subcutaneous glucagon. B. I.V. bolus of dextrose 50%. C. 15 to 20 g of a fast-acting carbohydrate such as orange juice. D. 10 U of fast-acting insulin.

C. 15 to 20 g of a fast-acting carbohydrate such as orange juice. (This client is having a hypoglycemic episode. Because the client is conscious, the nurse should first administer a fast-acting carbohydrate, such as orange juice, hard candy, or honey. For conscious patients able to take oral (PO) medications, readily absorbable carbohydrate sources (such as fruit juice) should be given. If the client has lost consciousness, the nurse should administer either I.M. or subcutaneous glucagon or an I.V. bolus of dextrose 50%.)

A male client with a history of hypertension is diagnosed with primary hyperaldosteronism. This diagnosis indicates that the client's hypertension is caused by excessive hormone secretion from which of the following glands? A. Adrenal medulla B. Pancreas C. Adrenal cortex D. Parathyroid

C. Adrenal cortex (Excessive secretion of aldosterone in the adrenal cortex is responsible for the client's hypertension. This hormone acts on the renal tubule, where it promotes reabsorption of sodium and excretion of potassium and hydrogen ions. Primary hyperaldosteronism (PA) is an underdiagnosed cause of hypertension. The classic presentation of PA includes hypertension and hypokalemia. However, in reality, most patients will present without hyperkalemia. The two primary causes are aldosterone-producing adenomas and bilateral adrenal hyperplasia of the zona glomerulosa.)

When instructing the female client diagnosed with hyperparathyroidism about diet, nurse Gina should stress the importance of which of the following? A. Restricting fluids B. Restricting sodium C. Forcing fluids D. Restricting potassium

C. Forcing fluids (The client should be encouraged to force fluids to prevent renal calculi formation. Drink enough fluids, mostly water, to produce nearly clear urine to lessen the risk of kidney stones. Avoid calcium-raising drugs. Certain medications, including some diuretics and lithium, can raise calcium levels. If taking such drugs, ask the doctor whether another medication may be appropriate.)

A male client has recently undergone surgical removal of a pituitary tumor. Dr. Wong prescribes corticotropin (Acthar), 20 units I.M. q.i.d. as a replacement therapy. What is the mechanism of action of corticotropin? A. It decreases cyclic adenosine monophosphate (cAMP) production and affects the metabolic rate of target organs. B. It interacts with plasma membrane receptors to inhibit enzymatic actions. C. It interacts with plasma membrane receptors to produce enzymatic actions that affect protein, fat, and carbohydrate metabolism. D. It regulates the threshold for water resorption in the kidneys.

C. It interacts with plasma membrane receptors to produce enzymatic actions that affect protein, fat, and carbohydrate metabolism. (Corticotropin interacts with plasma membrane receptors to produce enzymatic actions that affect protein, fat, and carbohydrate metabolism. CRH is released from the hypothalamus. CRH stimulates the anterior pituitary to release ACTH. ACTH acts on the adrenal cortex to release cortisol and androgens. The increase in cortisol provides a negative feedback system to then decrease the amount of CRH released from the hypothalamus.

Capillary glucose monitoring is being performed every 4 hours for a female client diagnosed with diabetic ketoacidosis. Insulin is administered using a scale of regular insulin according to glucose results. At 2 p.m., the client has a capillary glucose level of 250 mg/dl for which he receives 8 U of regular insulin. Nurse Vince should expect the dose's: A. Onset to be at 2 p.m. and its peak to be at 3 p.m. B. Onset to be at 2:15 p.m. and its peak to be at 3 p.m. C. Onset to be at 2:30 p.m. and its peak to be at 4 p.m. D. Onset to be at 4 p.m. and its peak to be at 6 p.m.

C. Onset to be at 2:30 p.m. and its peak to be at 4 p.m. (Regular insulin, which is a short-acting insulin, has an onset of 15 to 30 minutes and a peak of 2 to 4 hours. Because the nurse gave the insulin at 2 p.m., the expected onset would be from 2:15 p.m. to 2:30 p.m. and the peak from 4 p.m. to 6 p.m. Regular insulin is a medication used in the management of Diabetes Mellitus and hyperglycemia of a variety of etiologies. It is in the short-acting insulin class of drugs.)

For a diabetic male client with a foot ulcer, the physician orders bed rest, a wet-to-dry dressing change every shift, and blood glucose monitoring before meals and bedtime. Why are wet-to-dry dressings used for this client? A. They contain exudate and provide a moist wound environment. B. They protect the wound from mechanical trauma and promote healing. C. They debride the wound and promote healing by secondary intention. D. They prevent the entrance of microorganisms and minimize wound discomfort.

C. They debride the wound and promote healing by secondary intention. (For this client, wet-to-dry dressings are most appropriate because they clean the foot ulcer by debriding exudate and necrotic tissue, thus promoting healing by secondary intention. Treatment of diabetic foot ulcers should be systematic for an optimal outcome. The most important point is to identify if there is any evidence of ongoing infection, by obtaining a history of chills, fever, looking for the presence of purulence or presence of at least two signs of inflammation that includes, pain, warmth, erythema or induration of the ulcer.)

A female client with a history of pheochromocytoma is admitted to the hospital in an acute hypertensive crisis. To reverse hypertensive crisis caused by pheochromocytoma, nurse Lyka expects to administer: A. mannitol (Osmitrol) B. methyldopa (Aldomet) C. phentolamine (Regitine) D. felodipine (Plendil)

C. phentolamine (Regitine) (Pheochromocytoma causes excessive production of epinephrine and norepinephrine, natural catecholamines that raise the blood pressure. Phentolamine, an alpha-adrenergic blocking agent given by I.V. bolus or drip, antagonizes the body's response to circulating epinephrine and norepinephrine, reducing blood pressure quickly and effectively.)

Acarbose (Precose), an alpha-glucosidase inhibitor, is prescribed for a female client with type 2 diabetes mellitus. During discharge planning, nurse Pauleen would be aware of the client's need for additional teaching when the client states: A. "It's best if I take the drug with the first bite of a meal." B. "The drug makes my pancreas release more insulin." C. "I should never take insulin while I'm taking this drug." D. "If I have hypoglycemia, I should eat some sugar, not dextrose."

D. "If I have hypoglycemia, I should eat some sugar, not dextrose." (Acarbose delays glucose absorption, so the client should take an oral form of dextrose rather than a product containing table sugar when treating hypoglycemia. Acarbose is FDA approved for the treatment of adults with type 2 diabetes mellitus as an adjunct to diet only or diet and exercise, depending on the patient's health status.)

Which nursing diagnosis takes highest priority for a female client with hyperthyroidism? A. Risk for imbalanced nutrition: More than body requirements related to thyroid hormone excess. B. Risk for impaired skin integrity related to edema, skin fragility, and poor wound healing. C. Body image disturbance related to weight gain and edema. D. Imbalanced nutrition: Less than body requirements related to thyroid hormone excess.

D. Imbalanced nutrition: Less than body requirements related to thyroid hormone excess. (In the client with hyperthyroidism, excessive thyroid hormone production leads to hypermetabolism and increased nutrient metabolism. These conditions may result in a negative nitrogen balance, increased protein synthesis and breakdown, decreased glucose tolerance, and fat mobilization and depletion. This puts the client at risk for marked nutrient and calorie deficiency, making Imbalanced nutrition: Less than body requirements the most important nursing diagnosis. Options B and C may be appropriate for a client with hypothyroidism, which slows the metabolic rate.)

After undergoing a subtotal thyroidectomy, a female client develops hypothyroidism. Dr. Smith prescribes levothyroxine (Levothroid), 25 mcg P.O. daily. For which condition is levothyroxine the preferred agent? A. Euthyroidism B. Graves' disease C. Thyrotoxicosis D. Primary hypothyroidism

D. Primary hypothyroidism

After undergoing a subtotal thyroidectomy, a female client develops hypothyroidism. Dr. Smith prescribes levothyroxine (Levothroid), 25 mcg P.O. daily. For which condition is levothyroxine the preferred agent? A. Euthyroidism B. Graves' disease C. Thyrotoxicosis D. Primary hypothyroidism

D. Primary hypothyroidism (Levothyroxine is the preferred agent to treat primary hypothyroidism and cretinism, although it also may be used to treat secondary hypothyroidism. Oral levothyroxine is primarily indicated for the treatment of primary, secondary, and tertiary hypothyroidism. Primary hypothyroidism is when the problem occurs in the thyroid gland, with the most common cause being an autoimmune condition (Hashimoto thyroiditis) followed up by iatrogenic hypothyroidism (after thyroidectomy).)

felodipine (Plendil)

Felodipine, an antihypertensive agent, is available only in extended-release tablets and therefore doesn't reduce blood pressure quickly enough to correct hypertensive crisis. Felodipine is an agent in the dihydropyridine class of calcium channel blockers. Felodipine is FDA approved and indicated in the treatment of essential hypertension. Reduction in blood pressure lowers the risk of cardiovascular morbidity and mortality. (calcium channel blocker)

Acromegaly surgical management

Hypophysectomy -Monitor neurological response, postoperatively, hematoma, compressing, the optic nerve, peripheral vision -Teach to report postnasal, drip or increasing swallowing (csf leak) -avoid coughing, sneezing, and straining to stool

Levothyroxine L.E.V.O.

L - Life long E - Early morning V - Very Hyper O - Oh baby is Fine (not teratogenic)

Aldosterone secreted by

adrenal cortex

aldosterone produced in

adrenal cortex

Hashimoto's thyroiditis

an autoimmune disease in which the body's own antibodies attack and destroy the cells of the thyroid gland. which can result in Graves Disease.

Up to 80% of Addison's disease cases in the United States are caused by

an autoimmune response.

pituitary gland

the major endocrine gland. A pea-sized body attached to the base of the brain, the pituitary is important in controlling growth and development and the functioning of the other endocrine glands.

7. Which patient statement indicates to the nurse that additional instruction is needed for a patient with chronic syndrome of inappropriate antidiuretic hormone (SIADH)? a. "I should weigh myself daily and report sudden weight loss or gain." b. "I need to shop for foods low in sodium and avoid adding salt to food." c. "I need to limit my fluid intake to no more than 1 quart of liquids a day." d. "I should eat foods high in potassium because diuretics cause potassium loss."

b. "I need to shop for foods low in sodium and avoid adding salt to food." (Patients with SIADH are at risk for hyponatremia, and a sodium supplement may be prescribed. The other patient statements are correct and indicate successful teaching has occurred.)

A 45-year-old male patient with suspected acromegaly is seen at the clinic. To assist in making the diagnosis, which questions should the nurse ask? a.Have you had a recent head injury? b.Do you have to wear larger shoes now? с.Is there a family history of acromegaly? d.Are you experiencing tremors or anxiety?

b.Do you have to wear larger shoes now? (Acromegaly causes an enlargement of the hands and feet. Head injury and family history are not risk factors for acromegaly. Tremors and anxiety are not clinical manifestations of acromegaly.)

2. A patient is scheduled for transsphenoidal hypophysectomy to treat a pituitary adenoma. What should the nurse include in preoperative teaching? a. Cough and deep breathe every 2 hours postoperatively. b. Remain on bed rest for the first 48 hours postoperatively. c. Avoid brushing teeth for at least 10 days after the surgery. d. You will be positioned flat with a cervical collar after surgery.

c. Avoid brushing teeth for at least 10 days after the surgery. (To avoid disruption of the suture line, the patient should avoid brushing the teeth for 10 days after surgery. It is not necessary to remain on bed rest after this surgery. Coughing is discouraged because it may cause leakage of cerebrospinal fluid (CSF) from the suture line. The head of the bed should be elevated 30 degrees to reduce pressure on the sella turcica and decrease the risk for headaches. A cervical collar is not needed.)

The adrenal medulla secretes the

catecholamines (epinephrine, norepinephrine, dopamine)

thyrotoxicosis

condition caused by the exposure of body tissue to excessive levels of thyroid hormone. causes hypermetabolism and increased sympathetic nervous system activity.

Onset of acromegaly fast or slow?

gradual

pituitary gland secretes

growth hormone — which regulates growth thyroid stimulating hormone (TSH) — which tells the thyroid gland to make hormones prolactin — which controls breast milk production adrenocorticotrophic hormone (ACTH) — which tells the adrenal glands to make hormones follicle stimulating hormone (FSH) — which is involved in the reproductive system luteinizing hormone — which is also involved in the reproductive system

beta cells produce

insulin

methyldopa (Aldomet)

methyldopa is an antihypertensive agent available in parenteral form, it isn't effective in treating hypertensive emergencies. Methyldopa is a medication used in the management and treatment of hypertension. It is in the centrally acting anti-hypertensive class of drugs.

Your pituitary gland also stores and releases:

oxytocin — which is involved in childbirth and breastfeeding vasopressin — which helps control the amount of salt and water in your body

Hypokalemia causes

paralytic ileus and muscle weakness. Significant muscle weakness occurs at serum potassium levels below 2.5 mmol/L but can occur at higher levels if the onset is acute. (Similar to the weakness associated with hyperkalemia, the pattern is ascending in nature affecting the lower extremities, progressing to involve the trunk and upper extremities and potentially advancing to paralysis.)

The parathyroids secrete

parathyroid hormone. Its main function is the production and secretion of parathyroid hormone (PTH), a polypeptide hormone responsible for maintaining serum calcium homeostasis. (The parathyroid is comprised of 4 small glands embedded in the posterior aspect of the thyroid gland.

ACTH deficiency may be caused by

pituitary disease or suppression of the hypothalamic-pituitary axis because of the administration of exogenous corticosteroids.

thyroid gland

produces hormones that regulate metabolism, body heat, and bone growth

Hyponatremia is indicated by

weight loss, abdominal cramping, muscle weakness, headache, and postural hypotension. (Symptoms depend upon the degree and chronicity of hyponatremia. Patients with mild-to-moderate hyponatremia (greater than 120 mEq/L) or gradual decrease in sodium (greater than 48 hours) have minimal symptoms. Patients with severe hyponatremia (less than 120 mEq/L) or rapid decrease in sodium levels have multiple varied symptoms.)


संबंधित स्टडी सेट्स

Exam 4 Ch. 11, 19 (evolve chapters: 27, 28, 29, 34)

View Set

Frequently Asked Questions Teacher Interview

View Set

PEDS Practice: Chapter 23: Nursing Care of the Child With an Alteration in Tissue Integrity/Integumentary Disorder

View Set

Principle Accounting 7, 8, 9, 10, 11

View Set